50
Preparing for the AP ® U.S. Government and Politics Exam To Accompany American Government: Institutions and Policies, 16th Edition & Gateways to Democracy: An Introduction to American Government, Updated 4th Edition By James Q. Wilson, John J. Dilulio, Jr., Meena Bose, Matthew Levendusky John G. Geer, Richard Herrera, Wendy J. Schiller, and Jeffrey A. Segal Eugene T. Chase David G. Benson Edmond North High School, Cherry Creek High School, Edmond, Oklahoma Greenwood Village, Colorado Karen K. Waples Fletcher L. Woolsey Cherry Creek High School, Cherry Creek High School, Greenwood Village, Colorado Greenwood Village, Colorado Benwari Singh Cherry Creek High School, Greenwood Village, Colorado Australia • Brazil • Mexico • Singapore • United Kingdom • United States 5 AP ® is a trademark registered by the College Board, which is not affiliated with, and does not endorse, this product.

Preparing for the AP U.S. Government and Politics Exam · 2020-03-20 · multiple-choice portion of the AP ... Congress could pass legislation providing stronger “whistleblower”

  • Upload
    others

  • View
    2

  • Download
    0

Embed Size (px)

Citation preview

Page 1: Preparing for the AP U.S. Government and Politics Exam · 2020-03-20 · multiple-choice portion of the AP ... Congress could pass legislation providing stronger “whistleblower”

Preparing for the AP® U.S. Government and Politics Exam

To Accompany American Government: Institutions and Policies, 16th Edition

& Gateways to Democracy: An Introduction to American Government, Updated 4th Edition

By James Q. Wilson, John J. Dilulio, Jr., Meena Bose, Matthew Levendusky

John G. Geer, Richard Herrera, Wendy J. Schiller, and Jeffrey A. Segal

Eugene T. Chase David G. Benson Edmond North High School, Cherry Creek High School, Edmond, Oklahoma Greenwood Village, Colorado

Karen K. Waples Fletcher L. Woolsey Cherry Creek High School, Cherry Creek High School, Greenwood Village, Colorado Greenwood Village, Colorado

Benwari Singh Cherry Creek High School, Greenwood Village, Colorado

Australia • Brazil • Mexico • Singapore • United Kingdom • United States

5

AP® is a trademark registered by the College Board, which is not affiliated with, and does not endorse, this product.

Page 2: Preparing for the AP U.S. Government and Politics Exam · 2020-03-20 · multiple-choice portion of the AP ... Congress could pass legislation providing stronger “whistleblower”

Printed in the United States of America Print Number: 01 Print Year: 2018

© 2019 Cengage Learning Unless otherwise noted, all content is © Cengage ALL RIGHTS RESERVED. No part of this work covered by the copyright herein may be reproduced, transmitted, stored, or used in any form or by any means graphic, electronic, or mechanical, including but not limited to photocopying, recording, scanning, digitizing, taping, Web distribution, information networks, or information storage and retrieval systems, except as permitted under Section 107 or 108 of the 1976 United States Copyright Act, without the prior written permission of the publisher.

National Geographic Learning/Cengage Learning is pleased to

offer our college-level materials to high schools for Advanced Placement®, honors, and electives courses.

To contact your National Geographic Learning representative, please call us toll-free at 1-888-915-3276 or visit us at

http://ngl.cengage.com.

For permission to use material from this text or product, submit all requests online at www.cengage.com/permissions

Further permissions questions can be emailed to [email protected].

ISBN: 978-1-337-78877-9 Cengage 20 Channel Center Street Boston, MA 02210 USA Cengage is a leading provider of customized learning solutions with employees residing in nearly 40 different countries and sales in more than125 countries around the world. Find your local representative at: www.cengage.com. Cengage products are represented in Canada by Nelson Education, Ltd. To learn more about Cengage platforms and services, register or access your online learning solution, or purchase materials for your course, visit www.cengage.com.

AP® is a trademark registered by the College Board, which is not affiliated with, and does not endorse, this product.

Page 3: Preparing for the AP U.S. Government and Politics Exam · 2020-03-20 · multiple-choice portion of the AP ... Congress could pass legislation providing stronger “whistleblower”

CONTENTS iii

© 2019 Cengage Learning, Inc. May not be scanned, copied or duplicated, or posted to a publicly accessible website, in whole or in part.

CONTENTS

Preface ix

About the Authors x

Part I: Strategies for the AP® United States Government and Politics Exam 1 Preparing for the AP® Exam 3

What’s in This Book 3 Setting Up a Review Schedule 4 Before the Examination 4

Taking the AP® U.S. Government and Politics Exam 6 Navigating the Course Framework 6 Foundational Documents and Required Supreme Court Cases 7 Strategies for the Multiple-Choice Section 8 Types of Multiple-Choice Questions 9 Illustrate through Scenario 9 Application of Concept 9 Free-Response Questions 12 Scoring for Free-Response Questions 14

A Diagnostic Test 15 Scoring This Diagnostic Test 30 AP® Answer Key 30 Answers to the Multiple-Choice Questions 30 Answers to the Free-Response Questions 35 Calculating Your Score on the AP® Exam 40

Part II: A Review of AP® U.S. Government and Politics 41 Unit 1: Foundations of American Democracy 42

Chapter 1: The Articles of Confederation and the Constitution 42 Colonial Background 43 The Articles of Confederation 43 The Constitutional Convention 44 The Madisonian Model 45 Ratification of the Constitution 45 Altering the Constitution 46 Multiple Choice Questions 47 Free-Response Questions 49 Answers 50

iii

Page 4: Preparing for the AP U.S. Government and Politics Exam · 2020-03-20 · multiple-choice portion of the AP ... Congress could pass legislation providing stronger “whistleblower”

iv CONTENTS

© 2019 Cengage Learning, Inc. May not be scanned, copied or duplicated, or posted to a publicly accessible website, in whole or in part.

Chapter 2: Federalism 52 Systems of Government 53 The Constitutional Basis of Federalism 54 The Evolution of Federalism 55 Federalism Today 56 Multiple Choice Questions 57 Free-Response Questions 60 Answers 61

Chapter 3: Balancing Government Power and Individual Rights 63 Colonial Background 64 Political Life After the Revolution 65 What Government Does? 65 What Is Democracy? 65 Theories About How American Democracy Works 66 The Role of Individual Rights 66 Multiple Choice Questions 67 Free-Response Questions 69 Answers 70

Chapter 4: The Constitution and Policy Making 73 Origins of the Madisonian Model 74 Checks and Balances 75 The Madisonian Model in Modern America 75 Multiple Choice Questions 76 Free-Response Questions 78 Answers 79

Unit 2: Interactions Among Branches of Government 81 Chapter 5: The Legislative Branch 81

Congressional Basics 83 The Functions of Congress 84 The Powers of Congress 84 Congressional Elections 85 The Congressional Committee System 86 Congressional Leadership 86 House Leadership 87 Senate Leadership 87 The Congressional Legislative Process 87 Multiple Choice Questions 88 Free-Response Questions 91 Answers 92

Chapter 6: The Executive Branch 94 Presidential Basics 95 The Roles of the President 95 The President as Party Chief and Politician 97 Special Use of Presidential Power 98

Page 5: Preparing for the AP U.S. Government and Politics Exam · 2020-03-20 · multiple-choice portion of the AP ... Congress could pass legislation providing stronger “whistleblower”

CONTENTS v

© 2019 Cengage Learning, Inc. May not be scanned, copied or duplicated, or posted to a publicly accessible website, in whole or in part.

The Executive Branch 98 Multiple Choice Questions 99 Free-Response Questions 102 Answers 103

Chapter 7: The Judicial Branch 105 The Federal Court System 106 District Courts 107 Appellate Courts 107 The Supreme Court and How it Works 108 Judicial Nominations 109 Judicial Policy Making 109 Checks on Judicial Power 110 Multiple Choice Questions 111 Free-Response Questions 113 Answers 114

Chapter 8: The Bureaucracy 116 Bureaucratic Basics 117 The Organization of the American Bureaucracy 118 Staffing the Bureaucracy 119 Reforming the Bureaucracy 119 Bureaucratic Policy Making 120 Multiple Choice Questions 121 Free-Response Questions 124 Answers 124

Unit 3: Civil Liberties and Civil Rights 127 Chapter 9: The Concept of Civil Liberties 127

Freedom of Religion 129 Freedom of Expression 130 Peaceful Assembly and Petition 131 The Right to Privacy 131 Privacy and National Security 132 Cruel and Unusual Punishment 132 Multiple Choice Questions 132 Free-Response Questions 135 Answers 136

Chapter 10: Incorporating the Bill of Rights 139 Civil Liberties and Selective Incorporation 140 Rights of the Accused 141 The Right to Privacy and Incorporation 141 Multiple Choice Questions 142 Free-Response Questions 144 Answers 145

Page 6: Preparing for the AP U.S. Government and Politics Exam · 2020-03-20 · multiple-choice portion of the AP ... Congress could pass legislation providing stronger “whistleblower”

vi CONTENTS

© 2019 Cengage Learning, Inc. May not be scanned, copied or duplicated, or posted to a publicly accessible website, in whole or in part.

Chapter 11: Civil Rights 147 Civil Rights vs. Civil Liberties 148 African Americans and Civil Rights 149 The African American Civil Rights Movement 150 Women and Equal Rights 151 Latinos and Civil Rights 151 Affirmative Action 152 Other Civil Rights Movements 152 The Supreme Courts and Levels of Scrutiny 153 Multiple Choice Questions 153 Free-Response Questions 157 Answers 158

Chapter 12: Civil Rights and Government Implementation 160 The Issue of Enforcement 161 Turning Court Rulings into Legislation 161 Women’s Rights and the Issue of Interpretation 162 Voting Rights and the Measurement of Success 163 Multiple Choice Questions 164 Free-Response Questions 166 Answers 167

Chapter 13: The Courts and Minority Rights 169 The Concept of Minority Rights 170 The Role of the Courts 171 More Recent Courts and Shifting Opinions 171 Affirmative Action: The Modern Debate 172 Multiple Choice Questions 172 Free-Response Questions 175 Answers 176

Unit 4: American Political Ideologies and Beliefs 178

Chapter 14: Political Ideologies in America 178 What It Means to Be Liberal or Conservative 179 The Two Major Parties Today 180 Details of Economic Policy 181 Fiscal Policy 181 Monetary Policy 181 The Emergence of Libertarianism 182 Multiple Choice Questions 183 Free-Response Questions 186 Answers 187

Chapter 15: Political Socialization 189 Core Values in America 190 Political Socialization 191 Crises and Rally Events 192

Page 7: Preparing for the AP U.S. Government and Politics Exam · 2020-03-20 · multiple-choice portion of the AP ... Congress could pass legislation providing stronger “whistleblower”

CONTENTS vii

© 2019 Cengage Learning, Inc. May not be scanned, copied or duplicated, or posted to a publicly accessible website, in whole or in part.

Multiple Choice Questions 193 Free-Response Questions 196 Answers 196

Chapter 16: Polling and Public Opinion 199 Public Opinion and the Political Process 200 The Process of Polling 201 Multiple Choice Questions 203 Free-Response Questions 205 Answers 206

Unit 5: Political Participation 208 Chapter 17: The Decision to Vote 208

Expansion of Voting Rights 209 Voter Turnout 210 Voter Preferences 211 Structural Barriers to Voting 212 Models of Voting Behavior 212 Multiple Choice Questions 213 Free-Response Questions 216 Answers 217

Chapter 18: Political Parties and Interest Groups 220 Political Parties and American Politics 222 Party Organization 222 Why the Two-party System Endures 223 How Parties Grow and Change 223 Interest Groups and Why They Exist 224 Types of Interest Groups 225 Interest Group Success 226 The Competitive World of Government Lobbying 226 Multiple Choice Questions 227 Free-Response Questions 230 Answers 231

Chapter 19: Campaigns, Elections, and Campaign Finance 234 Selecting Candidates 236 Running for President 237 The Modern Campaign 237 Campaign Finance 238 The Electoral College 240 Multiple Choice Questions 241 Free-Response Questions 244 Answers 245

Chapter 20: The Media 247 Politics and Media Coverage 248 What the Media Chooses to Cover 249

Page 8: Preparing for the AP U.S. Government and Politics Exam · 2020-03-20 · multiple-choice portion of the AP ... Congress could pass legislation providing stronger “whistleblower”

viii CONTENTS

© 2019 Cengage Learning, Inc. May not be scanned, copied or duplicated, or posted to a publicly accessible website, in whole or in part.

Media’s Effects on the Electorate 249 Media as a Business 250 Multiple Choice Questions 251 Free-Response Questions 253 Answers 254

Part III: Practice Tests 256 Practice Test 1 257

Answer Key for Multiple-Choice Questions 271 Answers for Multiple-Choice Questions 271 Answers to Free-Response Questions 275 Calculating Your Score on the AP® Exam 278

Practice Test 2 279 Answer Key for Multiple-Choice Questions 293 Answers to Multiple-Choice Questions 293 Answers to Free-Response Questions 298 Calculating Your Score on the AP® Exam 300

Page 9: Preparing for the AP U.S. Government and Politics Exam · 2020-03-20 · multiple-choice portion of the AP ... Congress could pass legislation providing stronger “whistleblower”

ix © 2019 Cengage Learning, Inc. May not be scanned, copied or duplicated, or posted to a publicly accessible website, in whole or in part.

PREFACE

We hope this book helps you review all of the material covered in an AP® U.S. Government and Politics course and prepares you well for the AP® exam. AP® Government is an exciting and engaging course, and there is no substitute for what you learn in the classroom. However, some of the most important learning happens during review, when you can tie together all of the facts and concepts to see the big picture. The content in this book is condensed, but we are confident that it covers most of what you will see on the AP® exam.

Eugene T. Chase Edmond North High School, Edmond, Oklahoma

Karen K. Waples Cherry Creek High School, Greenwood Village, Colorado

Benwari Singh Cherry Creek High School, Greenwood Village, Colorado

David G. Benson Cherry Creek High School, Greenwood Village, Colorado

Fletcher L. Woolsey Cherry Creek High School, Greenwood Village, Colorado

Page 10: Preparing for the AP U.S. Government and Politics Exam · 2020-03-20 · multiple-choice portion of the AP ... Congress could pass legislation providing stronger “whistleblower”

x © 2019 Cengage Learning, Inc. May not be scanned, copied or duplicated, or posted to a publicly accessible website, in whole or in part.

ABOUT THE AUTHORS

EUGENE T. CHASE has taught social studies for more than 25 years. He is the Social Studies Department chair at Edmond North High School in Edmond, Oklahoma. At Edmond North, he teaches both U.S. and Comparative Government and Politics. A reader and table leader at the U.S. Government and Politics reading, he has presented workshops across the United States, including twice at the AP® National Conference. Mr. Chase was an author of the AP® Teacher’s Resource Guide accompanying American Government and Politics Today. He is also an author for another Fast Track to a 5.

KAREN K. WAPLES has taught since 1989. Coordinator of the Social Studies Department at Cherry Creek High School, she has taught three Advanced Placement courses—U.S. History, U.S. Government and Politics, and Comparative Government and Politics. In 1997, she received the Colorado Governor’s Award for Excellence in Education. A presenter at conferences nationwide, she is an author of two other review books in the Fast Track to a 5 series and several Teacher’s Resource Guides. Prior to teaching, Ms. Waples practiced law.

BENWARI SINGH, a teacher since 1992, is a member of the Cherry Creek High School Social Studies Department. Beginning with AP® U.S. History, he now teaches both AP® Comparative Government and Politics and AP® U.S. Government and Politics. A reader and now a table leader for AP® examination scoring, he has written two lessons in AP® Comparative Government and Politics for the College Board. Mr. Singh is an author of another Fast Track to a 5 and a Teacher’s Resource Guide to accompany The Challenge of Democracy.

DAVID G. BENSON has been teaching since 1979. A member of the Cherry Creek High School Social Studies Department, he comes to AP® U.S. Government and Politics by way of U.S. History, Humanities, Law, World History, International Relations, World Geography, and AP® Comparative Government and Politics. He has been an AP® examination reader for the College Board and is an author of another Fast Track to a 5.

FLETCHER L. WOOLSEY has been a teacher and debate coach since 2006. Starting in 2008, he joined the thriving professional community of AP® U.S. Government and Politics teachers at Cherry Creek High School, where he also teaches U.S. History and Sociology. He has worked as an AP® reader and has been placed in charge of adapting his school’s curriculum to the new AP® U.S. Government and Politics framework. In addition to this Fast Track, he also developed a new Teacher Resource Guide to accompany John Q. Wilson’s American Government, 16th Edition.

Page 11: Preparing for the AP U.S. Government and Politics Exam · 2020-03-20 · multiple-choice portion of the AP ... Congress could pass legislation providing stronger “whistleblower”

Part I

Strategies for the AP® United States Government and Politics Exam

Page 12: Preparing for the AP U.S. Government and Politics Exam · 2020-03-20 · multiple-choice portion of the AP ... Congress could pass legislation providing stronger “whistleblower”
Page 13: Preparing for the AP U.S. Government and Politics Exam · 2020-03-20 · multiple-choice portion of the AP ... Congress could pass legislation providing stronger “whistleblower”

3 © 2019 Cengage Learning, Inc. May not be scanned, copied or duplicated, or posted to a publicly accessible website, in whole or in part.

PREPARING FOR THE AP® EXAM Advanced Placement can be exhilarating. Whether you are taking an AP® course at your school or you are working on AP® independently, the stage is set for a great intellectual experience. As the school year progresses and you burrow deeper and deeper into the coursework, you will begin to make connections between the ideas and concepts that form the basis of government and politics in the United States. Fleshing out those concepts is exciting. More exciting still is recognizing examples of these concepts in the media. More importantly, this course prepares you to become a participatory citizen and an informed voter.

But sometime after New Year’s Day, when the examination begins to loom on a very real horizon, Advanced Placement can seem downright intimidating—in fact, offered the opportunity to take the examination for a lark, even adults long out of high school refuse. If you dread taking the test, you are in good company.

The best way to deal with an AP® examination is to master it, not let it master you. If you can think of these examinations as a way to show off how your mind works, you have a leg up: attitude does help. If you are not one of those students, there is still a lot you can do to sideline your anxiety. Focused review and practice time will help you master the examination so that you can walk in with confidence and get a 5.

WHAT’S IN THIS BOOK This book is keyed to American Government: Institutions and Policies, by James Q. Wilson, John J. Dilulio, Jr., Meena Bose, Matthew Levendusky, and Gateways to Democracy: An Introduction to American Government, by John G. Geer, Richard Herrera, Wendy J. Schiller, and Jeffrey A. Segal; but because it follows the College Board Course Framework, it is compatible with all textbooks. It is divided into three sections. Part I offers suggestions for getting yourself ready, from signing up to take the test and sharpening your pencils to organizing a free-response question. At the end of Part I you will find a Diagnostic Test. This test has all the elements of the U.S. Government and Politics examination, but the multiple-choice questions are organized according to the College Board Course Framework. When you go through the answers at the end of the Diagnostic Test, you will see how the questions are connected to the Course Framework. If there is one topic that is giving you trouble, it would be a good idea to review that topic. Page references at the end of each answer indicate where you will find the discussion on that particular point in both the 16th edition of American Government: Institutions and Policies and the updated 4th edition of Gateways to Democracy. Scoring is explained, so you will have some idea of how well you can do.

Part II, made up of twenty chapters—again following the College Board Course Framework—is especially valuable for those who took the course in the fall semester and are taking the examination months later, in the spring. Each chapter is connected to one of the Big Ideas and Enduring Understandings around which the Framework is built. These chapters are not a substitute for your textbook and class discussion; they simply review the U.S. Government and Politics course. At the end of each chapter you will find multiple-choice and free-response questions based on the material in that chapter. Again, you will find page references at the end of each question directing you to the discussion on that particular point in American Government: Institutions and Policies and Gateways to Democracy.

AP® is a trademark registered by the College Board, which is not affiliated with, and does not endorse, this product.

Page 14: Preparing for the AP U.S. Government and Politics Exam · 2020-03-20 · multiple-choice portion of the AP ... Congress could pass legislation providing stronger “whistleblower”

4 PREPARING FOR THE AP® EXAM

© 2019 Cengage Learning, Inc. May not be scanned, copied or duplicated, or posted to a publicly accessible website, in whole or in part.

Part III has two full-length practice AP® U.S. Government and Politics examinations. At the end of each test you will find the answers, explanations, and references to American Government: Institutions and Policies and Gateways to Democracy for the multiple-choice and free-response questions. Following the answers and explanations is a worksheet that you can fill in to calculate your score and give you an approximation of how you might score on the AP® exam.

SETTING UP A REVIEW SCHEDULE If you have been steadily doing your homework and keeping up with the course work, you are in good shape. But even if you’ve done all that—or if it’s too late to do all that—there are some more ways to get it all together. Read Part I of this book, which explains the format of the AP® exam and provides advice about how to prepare for it. You will be much more comfortable going into the test if you understand how the test questions are designed and how best to approach them. Then take the Diagnostic Test and see where you are right now.

Take out a calendar and set up a schedule for yourself. If you begin studying early, you can chip away at the review chapters in Part II. You’ll be surprised—and pleased—by how much material you can cover with half an hour a day for a month or so before the test. Look carefully at Big Idea or Learning Objective identified in the Diagnostic Test; if you missed a number of questions in one particular area, allow more time for the chapters that cover that area of the course. The practice tests in Part III will give you more experience with different kinds of multiple-choice questions and the wide range of free-response questions. If time is short, skip reading the review chapters. Look at the Key Terms and Key Concepts at the beginning of each chapter to be sure that you are familiar with the vocabulary and broad ideas you will find in the examination, and work on the multiple-choice and free-response questions at the end of each review. This will give you a good idea of your understanding of that particular topic. Then practice taking the test in Part III.

If time is really short, go straight from Part I to Part III: Taking practice tests multiple times is a great way to become more comfortable with the test and the types of questions you will need to answer.

BEFORE THE EXAMINATION Long before the AP® exam in May, you need to make sure that you are registered to take the test. Registration is handled a little differently at each school. Be sure to check with your teacher or the AP® coordinator to make sure that you are doing all that you need to do. If you need testing accommodations based on a 504 or an Individualized Educational Plan (IEP), you must let your school’s AP® coordinator know as soon as possible. The deadline to request accommodations is typically in early February. If your family cannot afford to pay the full testing fee, do not be shy and do not skip registering for the test. Inform your AP® coordinator. Reduced fees are available for those students who qualify. If you are studying AP® independently, call AP® Services at the College Board for the name of the local AP® coordinator, who will help you through the registration process.

The evening before the exam is not a great time for partying. Nor is it a great time for cramming. If you like, look over class notes or drift through your textbook, concentrating on the broad outlines, not the small details, of the course. You might also want to skim through this book and read the AP® tips.

The evening before the exam is a great time to get your things together for the next day. Sharpen a fistful of no. 2 pencils with good erasers for the multiple-choice section; set out several black or dark-blue ballpoint pens for the free-response questions; get a watch, as cell phones and smart watches are not allowed in the

Page 15: Preparing for the AP U.S. Government and Politics Exam · 2020-03-20 · multiple-choice portion of the AP ... Congress could pass legislation providing stronger “whistleblower”

PREPARING FOR THE AP® EXAM 5

© 2019 Cengage Learning, Inc. May not be scanned, copied or duplicated, or posted to a publicly accessible website, in whole or in part.

testing room; get a piece of fruit or a power bar and a bottle of water for the break; make sure you have your AP® ID number and whatever photo identification and admission tickets may be required. Then relax. And get a good night’s sleep.

On the day of the examination it is wise not to skip breakfast—studies show that students who eat a hot breakfast before testing get higher grades. Be careful not to drink a lot of liquids, necessitating a trip to the bathroom during the test. Breakfast will give you the energy you need to power through the test and the nutrition will keep your brain firing on all cylinders. You will spend some time waiting while everyone is seated in the right room for the right test, so prepare to be patient. Bring a high-energy snack to eat during the short break between Section I and Section II. The U.S. Government and Politics exam lasts for three hours, not counting breaks, so be prepared for a long morning. You do not want to be distracted by a growling stomach or hunger pangs.

Be sure to wear comfortable clothes, taking along a sweater in case the heating or air-conditioning is erratic. Be sure, too, to wear clothes you like—everyone performs better when they think they look better—and by all means wear your lucky socks. Now go get a 5.

Page 16: Preparing for the AP U.S. Government and Politics Exam · 2020-03-20 · multiple-choice portion of the AP ... Congress could pass legislation providing stronger “whistleblower”

6 © 2019 Cengage Learning, Inc. May not be scanned, copied or duplicated, or posted to a publicly accessible website, in whole or in part.

TAKING THE AP® U.S. GOVERNMENT AND POLITICS EXAM

The AP® U.S. Government and Politics examination consists of two sections: Section I has 55 multiple-choice questions; Section II has four free-response questions. You will have 80 minutes for the multiple-choice portion. There will be a short break between sections. You then have 100 minutes for the free-response portion. There will be four questions, each with a distinct question type. One question will be concept analysis, one asks you to interpret a chart or map, one will ask you about Supreme Court cases, and the last will ask you to write an argumentative essay. The College Board recommends that you spend 20 minutes on each of the first three questions and 40 minutes on the last, making for 100 minutes total.

NAVIGATING THE COURSE FRAMEWORK At some point in your study of AP® U.S. Government and Politics you will need to familiarize yourself with the Course Framework. It is readily available on the College Board website at www.collegeboard.org and can be found with a quick search for “AP U.S. government course framework.” The Framework divides the content of the course into five Big Ideas:

Constitutionalism The exam will expect you to be familiar with the Constitution, particularly with what various branches of the government can and cannot do. Knowing specific powers of government and how rights are interpreted and applied and a key part of the exam. Modern debates over the power of the president to regulate trade and immigration fall under this category.

Liberty and Order The course will continually return to the need to balance individual rights with the safety and stability of society. Examining what this looks like and how it effects life in the United States will be a frequent exercise. Attempting to balance the privacy and rights of citizens with the need to protect the country from terrorist threats is a contemporary example of this big idea.

Civic Participation in a Representative Democracy Voting is central to the American system of government. Understanding voting habits and encouraging political participation is a chief concern of the exam. Balancing Americans’ individualist tendencies with the need to participate in a democratic republic is a particularly common topic. Voting turnout is often low, raising fears about the legitimacy of elected officials. As voter turnout has steadily declined, this idea has become more and more of a concern.

Competing Policy-Making Interests Government action comes only through a mix of competition and cooperation between groups. Political parties are the obvious example, but the roles of interest groups, government agencies, and grassroots social movements will also play a major role. Modern government regulations regarding things like the environment and workplace safety are largely driven by the bureaucracy.

AP® is a trademark registered by the College Board, which is not affiliated with, and does not endorse, this product.

Page 17: Preparing for the AP U.S. Government and Politics Exam · 2020-03-20 · multiple-choice portion of the AP ... Congress could pass legislation providing stronger “whistleblower”

TAKING THE AP® U.S. GOVERNMENT AND POLITICS EXAM 7

© 2019 Cengage Learning, Inc. May not be scanned, copied or duplicated, or posted to a publicly accessible website, in whole or in part.

Methods of Political Analysis The AP® government exam is testing you to see whether you should earn political science credit at your future college or university. As such, the course will include delve into the actual practice of political science and analysis. This idea is intended to differentiate the course from simply a government-oriented history course. The explosion of social media and increasing reliance on polling data have driven a renewed interest in the field of political science.

Once you have a grasp of the five Big Ideas, you can move on to look at how the course is arranged. The Course Framework is divided into five units, each with a different mixture of Big Ideas:

• Unit 1: Foundations of American Democracy • Unit 2: Interactions Among Branches of Government • Unit 3: Civil Liberties and Civil Rights • Unit 4: American Political Ideologies and Beliefs • Unit 5: Political Participation

The questions on the AP® exam will be divided between the five units.

Within each unit, the content has been broken down further, to help you better understand what the AP® exam will expect you to know. The units are divided between three different types of content:

Enduring Understanding There will be three to five Enduring Understandings in each unit. These are very broad ideas that should become second nature to you by the end of the course. One of the AP® test designers described these as, “the things that you should still remember when you see your AP® government teacher in a restaurant ten years later.”

Learning Objectives Each Enduring Understanding will have between one and seven Learning Objectives. These are phrased as specific tasks that you should be able to complete at the end of the unit. A typical Learning Objective will read something like, “Describe linkage institutions.” These make great review questions for the days leading up to the exam.

Essential Knowledge These represent specific concepts and facts that you will be expected to know for the exam. Each multiple-choice question on the exam should be connected to a specific piece of Essential Knowledge. Review these along with the Learning Objectives to help prepare you for the exam.

FOUNDATIONAL DOCUMENTS AND REQUIRED SUPREME COURT CASES

The most recent redesign of the AP® U.S. Government and Politics Course Framework has made primary sources and Supreme Court cases significantly easier to study. The following nine documents are the readings required for the test, all of which are readily available online for free:

The U.S. Constitution (including the Bill of Rights and all other amendments) The Articles of Confederation The Declaration of Independence Brutus No. 1 (from the Antifederalist Papers) Federalist No. 10 Federalist No. 51 Federalist No. 70 Federalist No. 78 Letter From a Birmingham Jail

Page 18: Preparing for the AP U.S. Government and Politics Exam · 2020-03-20 · multiple-choice portion of the AP ... Congress could pass legislation providing stronger “whistleblower”

8 TAKING THE AP® U.S. GOVERNMENT AND POLITICS EXAM

© 2019 Cengage Learning, Inc. May not be scanned, copied or duplicated, or posted to a publicly accessible website, in whole or in part.

There are 15 required Supreme Court cases. For each case, you will need to know the issue before the Court (usually phrased as a question), the facts of the case, the Court’s ruling/holding, and the reasoning behind that ruling/holding. The following are the required Supreme Court cases for the exam:

Marbury v. Madison (1803) McCulloch v. Maryland (1819) Schenck v. United States (1919) Brown v. Board of Education (1954) Baker v. Carr (1961) Engel v. Vitale (1962) Gideon v. Wainwright (1963) Tinker v. Des Moines Independent School District (1969) New York Times Co. v. United States (1971) Wisconsin v. Yoder (1972) Roe v. Wade (1973) Shaw v. Reno (1993) United States v. Lopez (1995) McDonald v. Chicago (2010) Citizens United v. Federal Election Commission (2010)

STRATEGIES FOR THE MULTIPLE-CHOICE SECTION Here are some rules of thumb to help you work your way through the multiple-choice questions:

Answer every question. There are four possible answers for each question. Each correct answer is worth 1 point. There is no penalty for incorrect answers, so you should answer every question, even if you must guess. Some students like to go through the exam quickly and answer the questions they know, and then go back to the more difficult questions. If you do this, be sure you mark the correct line on the answer sheet and keep an eye on the time remaining for the test. When your 80 minutes are up, the exam will be taken from you. There will be no extensions, save for people with testing accommodations who have requested them ahead of time.

Read the question carefully. Pressured for time, many students make the mistake of reading the questions too quickly or merely skimming them. By reading a question carefully, you may already have some idea about the correct answer. You can then look for it in the responses. The AP® exam is designed to weed out great students from merely good ones. Questions can be deliberately tricky or deceiving, and the College Board is counting on you to miss them by rushing.

Eliminate any answer you know is wrong. You can write on the multiple-choice questions in the test book. As you read through the responses, draw a line through any answer you know is wrong.

Read all the possible answers, and then choose the most accurate response. AP® examinations are written to test your precise knowledge of a subject. Sometimes there are a few probable answers but one of them is more specific. For example, the Fifteenth Amendment gave the vote to people of all races, but only men could vote at that time. An answer that referred to black men would be more correct than an answer that referred to all people of all races.

Avoid absolute responses. These answers often include the words “always” or “never.” For example, the statement “Voter turnout in presidential elections is always slightly above 50 percent” is incorrect, because in the 1996 presidential election, turnout was a little below 50 percent. Absolute statements are usually a good clue that you are not looking at the correct answer.

Page 19: Preparing for the AP U.S. Government and Politics Exam · 2020-03-20 · multiple-choice portion of the AP ... Congress could pass legislation providing stronger “whistleblower”

TAKING THE AP® U.S. GOVERNMENT AND POLITICS EXAM 9

© 2019 Cengage Learning, Inc. May not be scanned, copied or duplicated, or posted to a publicly accessible website, in whole or in part.

TYPES OF MULTIPLE-CHOICE QUESTIONS There are various kinds of multiple-choice questions. You will notice that the phrase “Which of the following …” is used often. Here are some suggestions for how to approach each kind:

DEFINE, DESCRIBE, OR IDENTIFY QUESTIONS This is the most common type of multiple-choice question. It simply requires you to read the question and select the most correct answer. For example:

1. Which of the following describes a power of Congress, as listed in Article I of the Constitution? (A) Act as commander-in-chief of the military (B) Coining money (C) Implement laws passed by the executive branch (D) Operate public schools

ANSWER: B. Answer A is a power of the President, C is incorrect as the executive branch does not pass laws, and D is a power left to the states. The AP® exam is picky about phrases like “pass laws” in that they mean that very literally. The President may sign bills into law, but he or she does not pass them.

ILLUSTRATE THROUGH SCENARIO These questions will begin with a sentence or two that describes a scenario. It may be a real-life example or a hypothetical one. The scenario will set up a question that asks you to apply your knowledge of government and politics to determine the right answer. 1. The House Committee on Agriculture is concerned that a federally funded

program is not being implemented efficiently. In order to address the concerns the committee is most likely to take which of the following actions? (A) Recommend that funding for the program be cut or eliminated from the

federal budget. (B) Request that the President charge the cabinet with addressing the issues. (C) Conduct an oversight hearing to gather information about the agency’s

activities. (D) Begin impeachment proceedings to remove the Secretary of Agriculture.

ANSWER: C. A congressional committee is in charge of oversight, and such hearings are a big part of the power they wield over the bureaucracy. Don’t get too bogged down in the scenario itself. These questions are really quite similar to define, describe, or identify questions we just looked at.

APPLICATION OF CONCEPT These questions are essentially asking you to understand how the item/concept in question impacts the functioning of the American political system. 1. Which of the following is an effect of concurrent powers?

(A) It provides local government the opportunity to override federal laws. (B) It creates multiple access points for interest groups and citizens to change

policy. (C) It creates a more homogenous legal system. (D) It allows the President to govern from the bully pulpit.

ANSWER: B. Vesting power in various branches and levels of government means that citizens will have a variety of ways in which they can access government and thus impact the policies that government creates. Treat these as vocab questions, as they are testing you to see whether you know what a particular term means.

Page 20: Preparing for the AP U.S. Government and Politics Exam · 2020-03-20 · multiple-choice portion of the AP ... Congress could pass legislation providing stronger “whistleblower”

10 TAKING THE AP® U.S. GOVERNMENT AND POLITICS EXAM

© 2019 Cengage Learning, Inc. May not be scanned, copied or duplicated, or posted to a publicly accessible website, in whole or in part.

COMPARISON QUESTIONS These questions set up a comparison between two related concepts. They will appear on the exam as a table, with one column for each concept. Each answer corresponds to a pair of responses. To be correct, the items in BOTH columns must be accurate. This is particularly challenging because the AP® exam will typically have at least one column containing a correct answer in every pair. 1. Which of the following is an accurate comparison of liberal and conservative

views on government regulation of the economy?

ANSWER: D. This is the only option in which BOTH columns are accurate. Save a little extra time for these questions, as they can be very tricky. Notice that the conservative column for answers A and B are correct, but not the liberal. Likewise, the liberal column for answer C is correct, but not the conservative column.

QUALITATIVE ANALYSIS—VISUAL These questions require you to interpret a mostly graphic stimulus, such as a cartoon or map. The purpose of these questions is to test your ability to understand non-textual information and be able to connect it to concepts that you have learned in your study of government. There will almost always be multiple questions per cartoon/map.

Source: Toles/Andrews McMeel Syndication

Page 21: Preparing for the AP U.S. Government and Politics Exam · 2020-03-20 · multiple-choice portion of the AP ... Congress could pass legislation providing stronger “whistleblower”

TAKING THE AP® U.S. GOVERNMENT AND POLITICS EXAM 11

© 2019 Cengage Learning, Inc. May not be scanned, copied or duplicated, or posted to a publicly accessible website, in whole or in part.

1. Which of the following best represents the opinion expressed in the cartoon? (A) The Republican Party is best suited to reduce the level of debt in the

United States. (B) The Republican Party is not doing enough to control spending by the

government. (C) The Republican Party is seen as being too tight with fiscal policy. (D) The Republican Party has been ignoring opinion polling data regarding

their voters. ANSWER: B. The cartoon is implying that the Republican Party is ignoring the growth of government debt. Notice that the image on the screen is a bomb with a burning fuse. Any following questions will usually ask you to expand this meaning to other areas that you have studied, such as the media.

QUANTITATIVE ANALYSIS—TEXT As the name implies, these questions ask you to analyze some sort of chart or graph, examining the statistics at hand. One of the main keys to getting these questions correct is to remember that any time you are asked to interpret the graph your answer MUST be something that can be proven by the graph ONLY. You may know it is true. You may think it is common sense. However, if it is not proven in the graph it CANNOT be the correct answer. Once again, each chart or graph will probably have multiple questions attached to it.

1. Which of the following best explains the difference in voter turnout shown in

the table above? (A) North Carolina has a larger population of eligible voters than South

Carolina. (B) North Carolina has more African Americans, who are more likely to turn

out. (C) North Carolina has more lenient early voting rules, permitting more people

to vote. (D) North Carolina has more people who voted for the winning candidate.

2. Which of the following concepts explains why gaps in voter turnout and elections, such as those shown in the table, can exist in the first place? (A) Federalism (B) Separation of powers (C) Limited government (D) Free enterprise

ANSWERS: C and A. Notice for the first question that answers about African Americans, population size, and voter tendencies may all be correct, but they are totally unrepresented on the table. C is the only option that you can infer using only the information at hand. Federalism is represented in the table as it implies that different states have different election procedures. This type of state independence is critical to federalism.

QUALITATIVE ANALYSIS The last type of test question is the qualitative analysis question. This will be much like the other analysis questions, except this time you will be reading a passage from a primary or secondary source. The source may be from one of the required foundational documents found in the AP® Course Framework or it could be

Page 22: Preparing for the AP U.S. Government and Politics Exam · 2020-03-20 · multiple-choice portion of the AP ... Congress could pass legislation providing stronger “whistleblower”

12 TAKING THE AP® U.S. GOVERNMENT AND POLITICS EXAM

© 2019 Cengage Learning, Inc. May not be scanned, copied or duplicated, or posted to a publicly accessible website, in whole or in part.

something else entirely. Like the other analysis questions, you can expect to see multiple questions for each passage, likely starting with one that asks you to summarize or identify the theme or purpose of the passage.

By a faction, I understand a number of citizens, whether amounting to a majority or a minority of the whole, who are united and actuated by some common impulse of passion, or of interest, adversed to the rights of other citizens, or to the permanent and aggregate interests of the community. There are two methods of curing the mischiefs of faction: the one, by removing its causes; the other, by controlling its effects. There are again two methods of removing the causes of faction: the one, by destroying the liberty which is essential to its existence; the other, by giving to every citizen the same opinions, the same passions, and the same interests. It could never be more truly said than of the first remedy, that it was worse than the disease. Liberty is to faction what air is to fire, an aliment without which it instantly expires. But it could not be less folly to abolish liberty, which is essential to political life, because it nourishes faction, than it would be to wish the annihilation of air, which is essential to animal life, because it imparts to fire its destructive agency.

-James Madison, Federalist No. 10 1. Which of the following best describes Madison’s primary concern in the passage

above? (A) Government power can easily get out of control. (B) Limitations on certain rights are necessary in civil society. (C) Democracy is the most fair and equitable form of government. (D) The creation of factions can lead to trouble if unchecked.

2. Based on the passage above, which of the following groups would most likely fit with Madison’s vision of “factions”? (A) Racial groups (B) Age cohorts (C) Political parties (D) Religious congregations

Answers: D and C. The AP® exam will stress reading comprehension, and the test is written in such a way that you will want to slow down and carefully read the passage. For the above passage, the first question is intended to make sure that you grasp the concept of factions and why they matter. The second question then expands this and more directly applies it to the practice of political science.

FREE-RESPONSE QUESTIONS There are four mandatory free-response questions on the U.S. Government and Politics examination. Each question will come from a distinctly different type or category of question. When you are preparing for the exam and taking practice exams, remember that there is going to be one of each of these types on the exam.

Concept Application This is very similar to one of the multiple-choice question types. You will be given a scenario and asked to apply what you have learned to that scenario. For example, the scenario might be related to a public health problem or natural disaster. You would then be asked to provide possible answers that Congress or the President might take to address the problem. These questions will typically require you to have a good knowledge of government powers and how branches interact.

Quantitative Analysis This is another type that closely resembles some of the multiple-choice questions. These will present you with a chart, graph, or map and ask you to identify a trend or pattern contained therein. Once you have done that, you will likely be asked to relate that trend or information to certain political institutions, behaviors, or processes.

Page 23: Preparing for the AP U.S. Government and Politics Exam · 2020-03-20 · multiple-choice portion of the AP ... Congress could pass legislation providing stronger “whistleblower”

TAKING THE AP® U.S. GOVERNMENT AND POLITICS EXAM 13

© 2019 Cengage Learning, Inc. May not be scanned, copied or duplicated, or posted to a publicly accessible website, in whole or in part.

Supreme Court Case Comparison These questions will describe the facts, holding, and decision of a Supreme Court case and then ask you to compare it to one of the required Supreme Court cases. The question will tell you all you need to know about the first case, but you will be required to remember the required case.

Argument Essay This question will ask you to write a proper essay in response to an analytical question. Each of the other three types of questions will be broken down into discreet responses, typically labeled a, b, c, and so on. This question will ask you to write an essay with a proper thesis and body paragraph organization. You will also be asked to refer to foundational documents as evidence for your arguments.

The free-response portion of the exam is 100 minutes long. The College Board recommends that you spend approximately 20 minutes on each of the first three questions and 40 minutes on the final question. Be sure to time yourself whenever you practice free-response answers, as the time restriction can be the most challenging portion. Also, notice that the argumentative essay gets twice the recommended time as the others. This is a good clue that they expect this portion to be more thought-out and deliberate than the others.

VOCABULARY In answering the free-response questions, carefully read the question and do exactly what it asks. The first three questions will use very similar vocabulary when it comes time to ask you to perform a certain task. It is important to note the word choices used in the questions:

Define To state the meaning of a word or phrase or to give a specific example. For instance, if a question asks you to define “iron triangle,” the response is “An iron triangle is a close, advantageous relationship between an interest group, a congressional committee, and an administrative agency.” Definitions are usually just one sentence.

Identify To select a factor, a person, or an idea and give it a name. For instance, if a question asks you to identify one advantage of incumbency, one possible response is “One advantage of incumbency is the opportunity to do casework for constituents.”

Explain why/explain how To give a cause or reason. Explanations usually include the word “because.” For instance, if a question asks you why the ability to do casework gives incumbents an advantage, one possible response is “By doing casework, such as helping a constituent get her Social Security check, members of Congress are able to leave a favorable impression on members of their district. This increases their chances of reelection because they are able to get positive results for their constituents, who will vote for them in the next election.”

Page 24: Preparing for the AP U.S. Government and Politics Exam · 2020-03-20 · multiple-choice portion of the AP ... Congress could pass legislation providing stronger “whistleblower”

14 TAKING THE AP® U.S. GOVERNMENT AND POLITICS EXAM

© 2019 Cengage Learning, Inc. May not be scanned, copied or duplicated, or posted to a publicly accessible website, in whole or in part.

SCORING FOR FREE-RESPONSE QUESTIONS The first three questions will be scored in a similar fashion. You will get one point for every task that you are asked to complete. For example, a one-point task will look something like this:

(A) Identify one power that Congress might use to address the problem described in the scenario above.

A two-point task will look something like this:

(B) Describe a similarity or difference in how Democrats and Republicans handle government regulation of the economy, and explain how that might affect policy they seek to implement.

One point is awarded for describing the similarity or difference, and a second point is awarded for explaining how that might influence the policies each party chooses to support. Questions will typically only be worth three to five points, so make every one count.

The argumentative essay will be scored a bit differently, as there tends to be some identical tasks, regardless of the question itself. Every argumentative essay will ask you to do the following:

Articulate a defensible claim or thesis You will be given one point for starting your essay by taking an explicit position on the question at hand. It will be important that you not only take a stance but also do more than simply restate the question. AP® graders will be looking for a “because” statement. Writing something like, “The Constitution did not uphold the values of representative democracy” will not earn a point. Instead, you should write, “The Constitution did not uphold the values of representative democracy because it is so heavily reliant upon undemocratic institutions like the Electoral College.”

Evidence usage Every argumentative essay will ask you to use at least one foundational document and at least one other piece of evidence. You will receive one point for each piece of evidence that you include, though you will not receive more than two points if you include more than two sources.

Reasoning You receive one point for making the connection between your specific evidence and the argument you are making. This should be an easy point, you just have to remember to do it! It is designed to punish test-takers who simply drop evidence into the essay without thinking about how it relates to the argument at hand.

Opposing arguments. Your argumentative essay will close with addressing an opposing viewpoint. You will need to state any opposing argument and then concede it, refute it, or offer a rebuttal. Concession consists of you acknowledging the arguments validity and why it is valid. Refutation consists of disproving the argument or invalidating it. Rebuttal acknowledges the argument as valid but then counters with an explanation as to why it is not really a major concern.

Take time to familiarize yourself with these question types and practice answering them. The AP® exam will reward people who are familiar with these question formats and who can quickly and efficiently prepare their responses.

Page 25: Preparing for the AP U.S. Government and Politics Exam · 2020-03-20 · multiple-choice portion of the AP ... Congress could pass legislation providing stronger “whistleblower”

15 © 2019 Cengage Learning, Inc. May not be scanned, copied or duplicated, or posted to a publicly accessible website, in whole or in part.

A DIAGNOSTIC TEST This diagnostic test will you give some indication of how you might score on the multiple-choice portion of the AP® U.S. Government and Politics exam. Of course, the exam changes every year, so it is never possible to predict a student’s score with certainty. This test will also pinpoint strengths and weaknesses on the key content areas.

AP® UNITED STATES GOVERNMENT AND POLITICS

Section I: Multiple-Choice Questions

Time: 80 minutes, 55 questions

Directions: Select the best answer from the four options provided. Questions 1-3 refer to the graph below:

Source: Pew Research Center

1. Which of the following accurately describes the data shown in the graph? (A) Traditional news media is losing ground to social media. (B) TV news viewership tends to spike right after presidential elections. (C) Cable TV news has seen a recent surge in popularity. (D) Newspapers and magazines continue to lose readers.

2. Fox News, MSNBC, and CNN are all known to have a relatively biased and opinionated perspective of the news. Based on the data shown in the graph, which of the following is true? (A) Targeting narrow, specific audiences makes economic sense for networks. (B) Bias in the news media is seen as unethical and is unpopular. (C) Americans prefer a balanced, honest approach to reporting the news. (D) Presidential candidates prefer to talk to reporters only from biased sources.

AP® is a trademark registered by the College Board, which is not affiliated with, and does not endorse, this product.

Page 26: Preparing for the AP U.S. Government and Politics Exam · 2020-03-20 · multiple-choice portion of the AP ... Congress could pass legislation providing stronger “whistleblower”

16 A DIAGNOSTIC TEST

© 2019 Cengage Learning, Inc. May not be scanned, copied or duplicated, or posted to a publicly accessible website, in whole or in part.

3. Based on your knowledge of the media and its coverage of elections, what are the viewers in the chart most likely to see on the news during an election cycle? (A) In-depth investigations into the practices and inner-workings of each

campaign. (B) Equal time granted to both sides of a particular issue or race for office. (C) Speeches by candidates, presented in full and unedited. (D) Extensive discussion surrounding who is ahead in which poll or who is

“winning.”

Questions 4-5 refer to the graph:

Source: Pew Research Center

4. Based on the information in the chart, which of the following actions would voters most like to see Congress undertake? (A) Pass a law increasing the minimum standards for water quality in the

United States. (B) Sue companies that excessively pollute for contributing to climate change. (C) Negotiate a treaty with other developed nations to reduce carbon

emissions. (D) Set aside more lands for national parks and wildlife refuges

5. Protecting the environment would fall to the hands of the Environmental Protection Agency (EPA). The EPA is part of which of the following? (A) The executive branch (B) The bureaucracy (C) The judiciary branch (D) The legislature branch

Page 27: Preparing for the AP U.S. Government and Politics Exam · 2020-03-20 · multiple-choice portion of the AP ... Congress could pass legislation providing stronger “whistleblower”

A DIAGNOSTIC TEST 17

© 2019 Cengage Learning, Inc. May not be scanned, copied or duplicated, or posted to a publicly accessible website, in whole or in part.

Questions 6-8 refer to the graph below:

Source: Pew Research Center

6. Based on the information provided in the graph, which of the following is the primary difference between male and female voters in the United States? (A) Female voters are more likely to vote on election day. (B) Female voters are typically better educated than male voters. (C) Female voters are, on average, older than male voters. (D) Female voters are generally more liberal than male voters.

7. Based on the information presented in the graph, which group saw the largest increase in support for the Democratic Party from 1994-2017? (A) The Millennial Generation (B) Women (C) The Silent Generation (D) Those with a college degree

8. Based on the information in the graph, which of the following media strategies would make the most sense for the Democrats in 2017? (A) Television advertising campaigns on major news networks. (B) Staging media-friendly events at manufacturing plants and other blue-

collar jobs. (C) Detailed policy proposals promoted through social media. (D) Holding town hall meetings to focus on healthcare and social security for

the elderly.

Page 28: Preparing for the AP U.S. Government and Politics Exam · 2020-03-20 · multiple-choice portion of the AP ... Congress could pass legislation providing stronger “whistleblower”

18 A DIAGNOSTIC TEST

© 2019 Cengage Learning, Inc. May not be scanned, copied or duplicated, or posted to a publicly accessible website, in whole or in part.

Questions 9 & 10 refer to the cartoon below:

Source: Steve Greenberg/CartoonStock.com

9. Which of the following best describes the message expressed in the cartoon? (A) Checks and balances help keep the government in check. (B) The ideological make-up of the Supreme Court can greatly impact its

rulings. (C) The Supreme Court has been historically more conservative than liberal. (D) Justices should be elected in order to better protect democracy.

10. In the opinion of the cartoonist, which of the following constitutional principles is most under attack? (A) Representative democracy (B) Separation of powers (C) Express and concurrent powers (D) An independent, unbiased judiciary

Page 29: Preparing for the AP U.S. Government and Politics Exam · 2020-03-20 · multiple-choice portion of the AP ... Congress could pass legislation providing stronger “whistleblower”

A DIAGNOSTIC TEST 19

© 2019 Cengage Learning, Inc. May not be scanned, copied or duplicated, or posted to a publicly accessible website, in whole or in part.

Questions 11-13 refer to the graph below:

Source: Pew Research Center

11. Based on the information presented in the graph, which of the following most likely represents generational voting habits in 2017? (A) Millennials are the most likely to vote Democrat. (B) The majority of Silent Generation voters are men. (C) Generation X has the fastest growing collection of conservatives. (D) Baby Boomers will make up the largest share of voters in the next election

12. Based on the information in the graph, which of the following policies will become more popular as millennials become a larger percentage of the voting population? (A) A reduction in federal regulation of corporate tax policy. (B) A reduction in federal regulation of environmental protection. (C) A reduction in federal regulation of abortion access. (D) A reduction in federal regulation of investment banking.

13. The difference in political opinion illustrated in the graph stems from generations learning from different media sources, forming different peer groups, and living in different family structures. Which of the following terms identifies that learning process? (A) Acculturation (B) Political socialization (C) Party identification (D) Realignment

Page 30: Preparing for the AP U.S. Government and Politics Exam · 2020-03-20 · multiple-choice portion of the AP ... Congress could pass legislation providing stronger “whistleblower”

20 A DIAGNOSTIC TEST

© 2019 Cengage Learning, Inc. May not be scanned, copied or duplicated, or posted to a publicly accessible website, in whole or in part.

Questions 14 & 15 refer to the map below:

Source: Cengage Learning, Inc.

14. This map shows a distribution of electoral votes in the 2016 presidential election, with Donald Trump representing the Republican Party and Hillary Clinton representing the Democratic Party. Which of the following best describes the strategy used in the campaign? (A) Republicans tended to target a smaller geographic area than Democrats. (B) Democrats targeted a few big states while Republicans targeted many

smaller states. (C) Republicans completely controlled Southern states and Democrats

controlled the North. (D) Democrats were less concerned with winning the popular vote than

Republicans. 15. Electoral maps such as this one highlight one of the principal criticisms of using

the Electoral College. Which of the following represents that criticism? (A) States which split their electoral votes often go ignored by candidates. (B) It is too hard to predict how many electoral votes each state will receive. (C) Presidential elections ignore the popular vote, as it does not determine the

winner. (D) The candidate who wins the states with the largest populations always

wins the election. 16. Which of the following is an accurate comparison of pluralist democratic theory

and elite democratic theory?

Page 31: Preparing for the AP U.S. Government and Politics Exam · 2020-03-20 · multiple-choice portion of the AP ... Congress could pass legislation providing stronger “whistleblower”

A DIAGNOSTIC TEST 21

© 2019 Cengage Learning, Inc. May not be scanned, copied or duplicated, or posted to a publicly accessible website, in whole or in part.

17. Which of the following is an accurate comparison of Wisconsin v. Yoder (1972) and Engel v. Vitale (1962)?

18. Which of the following is an accurate comparison of the House and Senate?

19. Which of the following is an accurate comparison of liberal and conservative

economic ideologies?

Page 32: Preparing for the AP U.S. Government and Politics Exam · 2020-03-20 · multiple-choice portion of the AP ... Congress could pass legislation providing stronger “whistleblower”

22 A DIAGNOSTIC TEST

© 2019 Cengage Learning, Inc. May not be scanned, copied or duplicated, or posted to a publicly accessible website, in whole or in part.

20. Which of the following is an accurate comparison of arguments for and against the program known as affirmative action?

Questions 21-24 refer to the below passage: Then it occurred to us that Birmingham's mayoral election was coming up in March, and we speedily decided to postpone action until after election day. When we discovered that the Commissioner of Public Safety, Eugene "Bull" Connor, had piled up enough votes to be in the run off, we decided again to postpone action until the day after the run off so that the demonstrations could not be used to cloud the issues. Like many others, we waited to see Mr. Connor defeated, and to this end we endured postponement after postponement. Having aided in this community need, we felt that our direct action program could be delayed no longer.

- Dr. Martin Luther King Jr., Letter from a Birmingham Jail 21. Which of the following best describes Dr. King’s main argument in this passage?

(A) Racism is destroying equality of opportunity for African Americans. (B) Protests and demonstrations were necessary following the mayoral

election. (C) The police brutality directed at African Americans is unacceptable. (D) Dr. King’s imprisonment is a violation of the Fourteenth Amendment’s

due process clause. 22. Dr. King brings up the issue of voting and hints at potential manipulation of

voting rights. Which of the following were often used as a way to deny African Americans’ voting rights until passage the Voting Rights Act of 1965? (A) Poll taxes (B) Closed primaries (C) Higher voting ages for African Americans (D) Literacy tests

23. Which of the following constitutional provisions specifically bans the type of racial discrimination in voting with which Dr. King is concerned? (A) The free exercise clause (B) The Fifteenth Amendment (C) The necessary and proper clause (D) The Great Compromise

24. The Supreme Court has acknowledged these concerns about African American voting rights but has also noted that protecting those rights cannot come at the expense of the rights of white voters. This precedent is related to which of the following cases? (A) Shaw v. Reno (1993) (B) Citizens United v. Federal Election Commission (2010) (C) United States v. Lopez (1995) (D) Brown v. Board of Education (1954)

Page 33: Preparing for the AP U.S. Government and Politics Exam · 2020-03-20 · multiple-choice portion of the AP ... Congress could pass legislation providing stronger “whistleblower”

A DIAGNOSTIC TEST 23

© 2019 Cengage Learning, Inc. May not be scanned, copied or duplicated, or posted to a publicly accessible website, in whole or in part.

Questions 25-27 refer to the passage below: From time to time our national history has been marred by forgetfulness of the Jeffersonian principle that restraint is at the heart of liberty. In 1789 the Federalists adopted Alien and Sedition Acts in a shabby political effort to isolate the Republic from the world and to punish political criticism as seditious libel. In 1865 the Radical Republicans sought to snare private conscience in a web of oaths and affirmations of loyalty…In our own times, the old blunt instruments have again been put to work. The States have followed in the footsteps of the Federalists and have put Alien and Sedition Acts upon their statute books. An epidemic of loyalty oaths has spread across the Nation until no town or village seems to feel secure until its servants have purged themselves of all suspicion of non-conformity by swearing to their political cleanliness.

-Source: John F. Kennedy, The Portsmouth Herald, April 4, 1961 25. Which of the following is most consistent with the author’s argument in the

passage? (A) American political history is not as free as many people seem to think. (B) Racial discrimination has a long history in the United States and needs to

stop. (C) The First Amendment is under attack and the President must defend it. (D) Civil liberties are vital to America, even though they are often trampled or

challenged. 26. Which of the following has most often been used to justify legislative acts such

as those Kennedy mentions? (A) Public safety (B) Government stability (C) Military power (D) Protection of democracy

27. Based on the passage, which of the following statements would Kennedy support? (A) The death penalty qualifies as cruel and unusual punishment. (B) Privacy is a right implied by the Bill of Rights. (C) Racial discrimination warrants direct government intervention. (D) Competition between parties breeds mistrust and laws that threaten liberty.

28. Which of the following illustrates Congress’ power of the purse? (A) Cutting the budget of a bureaucratic agency that it finds ineffective. (B) Reducing taxes on the wealthy to win over wealthy voters. (C) Ordering the Fed to print more money to stimulate the economy. (D) Passing campaign finance legislation to make elections more democratic.

29. The Framers of the Constitution understood that an amendment process was necessary to ensure that the document could change with the nation. Which of the following best describes the amendment process on which they settled? (A) A quick process that allows Congress to react swiftly to change. (B) A thorough process that requires cooperation of all three branches of

government. (C) A democratic process that requires voter approval of any proposed

amendment. (D) A slow process that attempts to balance state and federal power.

30. Which of the following identifies the constitutional significance of McDonald v. Chicago (2010)? (A) It restricted the expansion of federal power through the necessary and

proper clause. (B) It applied the Second Amendment to the states via selective incorporation. (C) It recognized the death penalty as cruel and unusual punishment. (D) It clarified and guaranteed defendants’ rights while in police custody.

Page 34: Preparing for the AP U.S. Government and Politics Exam · 2020-03-20 · multiple-choice portion of the AP ... Congress could pass legislation providing stronger “whistleblower”

24 A DIAGNOSTIC TEST

© 2019 Cengage Learning, Inc. May not be scanned, copied or duplicated, or posted to a publicly accessible website, in whole or in part.

31. The Supreme Court has just issued a ruling relating to discrimination in the workplace. The Court found in favor of the workers, citing its need to ensure due process for all citizens. This decision falls under which of the following? (A) Civil rights (B) Rights of the accused (C) Civil liberties (D) Equality of outcome

32. The Fourteenth Amendment has factored heavily in many anti-discrimination cases due to which of the following? (A) The right to freedom of association (B) The right to privacy (C) The free exercise clause (D) The equal protection clause

33. Congress has just appropriated $10 billion dollars to help fund improvements in public education. The money will be awarded to states with the only requirement being that the money has to somehow go to public education. Which of the following describes how this money is being distributed? (A) Federal mandates (B) Block grants (C) Revenue sharing (D) Devolution

34. Election night predictions typically rely heavily on exit polls, which are most often criticized for which of the following things? (A) Asking leading questions (B) Low margins of error (C) Intentional political bias (D) Poorly selected samples

35. Which of the following describes stare decisis? (A) A court ruling on a case using existing legal precedent. (B) Interpreting a case using only the black-and-white literal wording of the

Constitution. (C) A ruling issued without explanation that sets no precedent. (D) Refusing to hear an appeal due to a lack of standing to sue.

36. Which of the following was accomplished by the Declaration of Independence? (A) Establishing a concept of popular sovereignty. (B) Stipulating the freedoms and liberties protected in the United States. (C) Outlining a system of government for a new nation. (D) Explaining the need for a new Constitution.

37. The original system of government set up following the Revolutionary War funded itself largely through the sale of government land. This situation persisted because that original government lacked which of the following powers? (A) Borrowing from foreign nations (B) Taxation of citizens (C) Federal supremacy (D) Ratifying amendments

38. The decision in Marbury v. Madison (1803) was largely necessary due to the vagueness of which of the following parts of the Constitution? (A) Article III (B) The Necessary and Proper Clause (C) Article V (D) The Tenth Amendment

Page 35: Preparing for the AP U.S. Government and Politics Exam · 2020-03-20 · multiple-choice portion of the AP ... Congress could pass legislation providing stronger “whistleblower”

A DIAGNOSTIC TEST 25

© 2019 Cengage Learning, Inc. May not be scanned, copied or duplicated, or posted to a publicly accessible website, in whole or in part.

39. An interest group representing corporations that make weapons for the armed services has developed strong connections with high-ranking military personnel as well as many members of the Senate Armed Forces Committee. This alliance is best described as which of the following? (A) A caucus (B) A constituency (C) A corrupt bargain (D) An iron triangle

40. Title IX of the Education Amendments Act of 1972 deals with which of the following issues? (A) Discrimination on the basis of race. (B) Discrimination on the basis of gender. (C) Discrimination on the basis of sexual orientation. (D) Discrimination on the basis of disabilities.

41. Which of the following accurately describes the process of impeaching the President? (A) The House must vote to impeach a president by a two-thirds vote, and the

president is tried in the Senate, which must vote to convict by a three-fourths vote.

(B) The House must vote to impeach a president by a majority vote, and the president is tried in the Senate, which must vote to convict by a two-thirds vote.

(C) Both houses of Congress must vote on impeachment charges, and the Supreme Court has original jurisdiction to hear the case.

(D) The House votes on articles of impeachment, a trial is held in the Senate, and then the case is sent back to the House for a final vote on conviction.

42. Which of the following accurately describes most Americans’ political beliefs as they age? (A) Their beliefs remain largely the same, they just become more certain in

them. (B) The influence of peers and family declines and the influence of the media

increases. (C) Most Americans tend to become slightly more conservative as they age. (D) Ideological gaps between men and women grow larger with age.

43. The Pendleton Civil Service Act of 1883 sought to ensure that employees within the federal bureaucracy were hired and promoted based purely on their abilities. This became known as which of the following? (A) Credit claiming (B) Patronage (C) Civil service (D) The merit system

44. In the United States, candidates for political office are most often recruited by which of the following? (A) Political parties (B) Wealthy and powerful individuals (C) Current office holders (D) Members of the bureaucracy

45. Which of the following is the purpose of Congress using its oversight power? (A) Reviewing actions of the president (B) Reviewing actions of the bureaucracy (C) Reviewing a decision made by the Supreme Court (D) Reviewing the constitutionality of a law

Page 36: Preparing for the AP U.S. Government and Politics Exam · 2020-03-20 · multiple-choice portion of the AP ... Congress could pass legislation providing stronger “whistleblower”

26 A DIAGNOSTIC TEST

© 2019 Cengage Learning, Inc. May not be scanned, copied or duplicated, or posted to a publicly accessible website, in whole or in part.

46. Which of the following describes judicial review? (A) The right of the Supreme Court to rewrite a federal law to make it

constitutional. (B) The right of the Supreme Court to review the evidence in a trial and hear

new testimony. (C) The right of the Supreme Court to hear appeals in cases involving

constitutional issues. (D) The right of the Supreme Court to declare a law void if it violates the

Constitution. 47. Which of the following is a major determinant of whether a particular interest

group succeeds? (A) Whether or not the President likes the group in question. (B) The amount of economic resources the group has. (C) Having an endorsement from one of the major parties. (D) Receiving only favorable media attention.

48. In the 2010 mid-term elections, American voters gave Republicans a large majority in the House of Representatives and 49 seats in the Senate. Democrat Barack Obama was still President. Which of the following would be a likely outcome for this divided government? (A) The inability of the Federal Reserve to function. (B) A “co-leadership” system for congressional committees. (C) Delays in judicial confirmation. (D) The inability of the House to select a Speaker.

49. Which of the following describes the role of a Political Action Committee (PAC)? (A) A fundraising group that supports certain candidates and issues. (B) A consulting group that specializes in political strategy for a campaign. (C) A casual group of like-minded lawmakers who meet to discuss issues they

care about. (D) A voting group that attempts to boost awareness and voter turnout in their

area. 50. In McCulloch v. Maryland (1819) the Supreme Court issued which of the

following holdings? (A) There were strict limits to what could be considered interstate commerce. (B) Segregation was allowable if it was “separate but equal.” (C) State laws were secondary to federal law and the Constitution. (D) The Fourteenth Amendment did not apply to voting rights.

51. The federal government has passed multiple laws relating to the enforcement of federal immigration law. In some cities and states, the local law enforcement has been eager to help enforce this law, while in other cities and states, they have flatly refused to do what they see as the federal government’s job. This difficulty illustrates a frustration resulting from which of the following principles? (A) Majority rule, minority rights (B) Checks and balances (C) Popular rule (D) Federalism

52. Which of the following economic theories emphasizes deregulation and tax cuts? (A) Keynesianism (B) Supply-side economics (C) Monetarism (D) Fiscal federalism

Page 37: Preparing for the AP U.S. Government and Politics Exam · 2020-03-20 · multiple-choice portion of the AP ... Congress could pass legislation providing stronger “whistleblower”

A DIAGNOSTIC TEST 27

© 2019 Cengage Learning, Inc. May not be scanned, copied or duplicated, or posted to a publicly accessible website, in whole or in part.

53. Which of the following is responsible for setting monetary policy? (A) Congress and the President (B) Office of Management and Budget (C) Treasury Department (D) Federal Reserve

54. Which of the following describes one of the major concerns with the amount of attention paid to public polling? (A) Polls tend to guide too much of public opinion and media attention. (B) There is a shortage of polling companies, meaning many important polls

are not taken. (C) Most polls ask leading or misguided questions. (D) Even professional polls are rarely accurate.

55. President Franklin D. Roosevelt died in 1945, shortly after the start of his fourth term as President. Which of the following ensures that no person shall ever hold that many terms again? (A) The Tenure of Office Act (B) The Twenty-second Amendment (C) The holding in Baker v. Carr (1961) (D) Executive Order 9066

Page 38: Preparing for the AP U.S. Government and Politics Exam · 2020-03-20 · multiple-choice portion of the AP ... Congress could pass legislation providing stronger “whistleblower”

28 A DIAGNOSTIC TEST

© 2019 Cengage Learning, Inc. May not be scanned, copied or duplicated, or posted to a publicly accessible website, in whole or in part.

AP® UNITED STATES GOVERNMENT AND POLITICS

Section II: Free-Response Questions

Time: 140 minutes

Directions You have 140 minutes to answer all four of the following questions. Unless the directions indicate otherwise, respond to all parts of each question. It is recommended that you take a few minutes to plan and outline each answer. Spend approximately 20 minutes on each of the first three questions and 40 minutes on the argumentative essay. In your response use substantive examples where appropriate. 1. In 2017 the United State saw the rise of what came to be known as the #MeToo

movement, focusing on the prevalence of sexual assault and sexual harassment (primarily against women) in America. Advocates claim that many victims of these crimes do not come forward two main reasons. First and foremost, they assert that many victims feel as though little to nothing will be done to their attackers; the attackers will go lightly punished, if punished at all. Second, may victims fear retribution if their attacker is someone powerful or influential. These powerful people know this and thus proceed with their behavior, confident that the victim will take no action. After reading the scenario, respond to a, b, and c below. (a) Describe one action Congress could take to correct some of the problems

highlighted by the #MeToo movement. (b) In the context of the scenario, explain how the congressional action

described in Part A could be helped or hindered by Congress’ current relationship with the President.

(c) In the context of this scenario, explain how the #MeToo movement might impact the next election cycle and the ideologies of the two major parties.

Source: Pew Research Center

Page 39: Preparing for the AP U.S. Government and Politics Exam · 2020-03-20 · multiple-choice portion of the AP ... Congress could pass legislation providing stronger “whistleblower”

A DIAGNOSTIC TEST 29

© 2019 Cengage Learning, Inc. May not be scanned, copied or duplicated, or posted to a publicly accessible website, in whole or in part.

2. Use the graph on the preceding page to answer the questions. (a) Describe what most Americans believe about the status of immigrants who

come to the United States illegally. (b) Describe what impact this polling data might have on ONE of the major

party’s (Democrat or Republican) platforms in an upcoming election. (c) Explain how TWO of the following might influence the reliability of the

numbers seen here: i. Sample size

ii. Margin of error iii. Guiding questions

3. Alfred Smith and Galen Black were two Native American men working for a drug rehabilitation clinic in Oregon. As a condition of employment, the clinic required its employees to submit to regular drug testing. Smith and Black both failed a drug test when they tested positive for the drug peyote, which was illegal in Oregon at the time. Their firing not only cost them their jobs, but also made them ineligible for unemployment benefits from the state. Both men claimed that peyote had been used as part of a Native American religious ceremony. They claimed that the state’s categorization of their employment status was a violation of their constitutional rights.

In a 5-4 decision, the Supreme Court held in Employment Division v. Smith (1990) that no such rights violation had taken place. The majority of the Court asserted that Oregon’s law applied to everyone, not just members of the religion, and thus was not aimed at inhibiting one particular group. Likewise, the prohibition of peyote was determined not to excessively inhibit or critically alter the practice of the religion in question.

(a) Identify the specific constitutional clause that is common to both Employment Division v. Smith (1990) and Wisconsin v. Yoder (1972).

(b) Based on the constitutional clause identified in Part A, explain why the facts of Smith led to a different holding than that of the Court in Yoder.

(c) Describe a specific action that Native American interest groups might take in Oregon to legalize peyote and avoid this problem in the future.

4. Develop an argument that explains whether or not the use of judicial activism by the Supreme Court oversteps the intended power of the judicial branch. In your essay, you must: - Articulate a defensible claim or thesis that responds to the prompt and

establishes a line of reasoning - Support your claim with at least TWO pieces of accurate and relevant

information: o At least ONE piece of evidence must be from the United States

Constitution o Use a second piece of evidence from one of the following documents

Federalist No. 10 Federalist No. 51 Federalist No. 78

- Use reasoning to explain why your evidence supports your claim/thesis - Respond to an opposing or alternative perspective using refutation,

concession, or rebuttal

END OF EXAMINATION

Page 40: Preparing for the AP U.S. Government and Politics Exam · 2020-03-20 · multiple-choice portion of the AP ... Congress could pass legislation providing stronger “whistleblower”

30 A DIAGNOSTIC TEST

© 2019 Cengage Learning, Inc. May not be scanned, copied or duplicated, or posted to a publicly accessible website, in whole or in part.

SCORING THIS DIAGNOSTIC TEST Using the answer key below, score your test. Determine how many you answered correctly and how many you answered incorrectly.

1. C 2. A 3. D 4. A 5. B 6. D 7. D 8. C

9. B 10. D 11. A 12. C 13. B 14. B 15. C 16. C

17. A 18. D 19. C 20. B 21. B 22. D 23. B 24. A

25. D 26. A 27. B 28. A 29. D 30. B 31. A 32. D

33. B 34. D 35. A 36. A 37. B 38. A 39. D 40. B

41. B 42. B 43. D 44. A 45. B 46. D 47. B 48. C

49. A 50. C 51. D 52. B 53. D 54. A 55. B

Your score for the multiple-choice section will give you some idea of how well you might do on the AP® exam. Keep in mind that your answers for the free-response questions count for 50% percent and also determine your score. The practice multiple-choice test consists of 55 questions. To calculate your percentage, divide the number of questions you answered correctly by 55. The following is a rough estimate of how you might do on the AP® Exam:

Potential Score 88-100% 5 73-87% 4 60-72% 3 45-59% 2 Below 59% 1

AP® ANSWER KEY The AP® exam is built from the framework discussed at the beginning of this book. This answer key will not only explain the answer but will direct you toward which Learning Objective the question came from. Every question on the AP® exam should be connected to a Learning Objective and a piece of Essential Knowledge. This means that you should be able to use the framework as a study guide for the exam itself.

ANSWERS TO THE MULTIPLE-CHOICE QUESTIONS: 1. (C) PRD-3: The graph clearly shows an upswing in cable viewership right

around 2016 (American Government, 16th ed., p. 264/Gateways to Democracy, 4th ed., pp. 224-226).

2. (A) PRD-3: This answer describes the process known as “narrowcasting,” which networks take on as a way to secure a small but very loyal viewer base (American Government, 16th ed., p. 274/Gateways to Democracy, 4th ed., pp. 230-231).

3. (D) PRD-3: This answer describes horse race journalism, which is the strong tendency of media companies to focus on who is ahead in polls or elections rather than on substantive issues (American Government, 16th ed., p. 277/Gateways to Democracy, 4th ed., pp. AP25-AP26).

4. (A) CON-4: The chart clearly shows that the most popular answer was that the federal government was doing too little to protect water quality. This would imply that legislation protecting water sources would be quite popular (American Government, 16th ed., pp. 441-442/Gateways to Democracy, 4th ed., pp. 570-574).

Page 41: Preparing for the AP U.S. Government and Politics Exam · 2020-03-20 · multiple-choice portion of the AP ... Congress could pass legislation providing stronger “whistleblower”

A DIAGNOSTIC TEST 31

© 2019 Cengage Learning, Inc. May not be scanned, copied or duplicated, or posted to a publicly accessible website, in whole or in part.

5. (B) PMI-2: The EPA is an independent regulatory agency within the bureaucracy. The President appoints its leader and Congress can pass laws related to it, but it is considered part of the bureaucracy (American Government, 16th ed., pp. 366, 370, and 445-450/Gateways to Democracy, 4th ed., pp. 493-495 and 570-574).

6. (D) MPA-3: The graph clearly illustrates the gender gap, the term used to describe the tendency for women to vote noticeably more liberal than men (American Government, 16th ed., pp. 158-159/Gateways to Democracy, 4th ed., pp. 203-204).

7. (D) MPA-3: Getting this answer correct is all about looking at the correct portion of the data. You need to examine the starting % for each line and then compare it to the final %. College degree holders started out at 42% liberal and wound up at 58% liberal, a swing of 16 points (American Government, 16th ed., pp. 158-163/Gateways to Democracy, 4th ed., pp. 207-208).

8. (C) PMI-5: The graph shows that Democrats tend to be young, college educated, and female. Detailed, intelligent articles would appeal to crowds with higher educational attainment. (American Government, 16th ed., pp. 158–163/Gateways to Democracy, 4th ed., pp. 195-196 and 202-208).

9. (B) CON-4: The cartoon displays the Court with a noticeable rightward slant, indicating concerns that the conservative leanings of the justices will create rulings more guided by ideology than what is constitutionally correct (American Government, 16th ed., p. 337 Gateways to Democracy, 4th ed., pp. 541-544).

10. (D) PMI-1: The cartoonist is clearly concerned about the ideological shift in the Court and its impact on its rulings. The role of political ideologies may make the judiciary less independent than the Constitution intended. The Court should be the most unbiased of all branches (American Government, 16th ed., pp. 42-45/Gateways to Democracy, 4th ed., pp. 541-544).

11. (A) PMI-4: This question requires you to know that liberal outlooks are most commonly associated with the Democratic party. The millennials have the largest swath of liberal voters out of the four generations (American Government, 16th ed., pp. 158-163/Gateways to Democracy, 4th ed., pp. 202-208).

12. (C) PMI-4: While all answers should do with reducing regulation, a reduction in regulations relating to abortions is the only option popular with liberals. Most millennials are liberal or mostly liberal (American Government, 16th ed., pp. 140 and 195/ Gateways to Democracy, 4th ed., pp. 202-208).

13. (B) MPA-1: Political socialization is the term used to describe the process by which people come to acquire their political beliefs and attitudes (American Government, 16th ed., pp. 156-158/Gateways to Democracy, 4th ed., pp. 188-189).

14. (B) PRD-2: The map shows that Republicans clearly won more states, but Democrats won many of the largest states, with Texas and Florida being notable exceptions (American Government, 16th ed., pp. 335–336/Gateways to Democracy, 4th ed., pp. 336-343).

15. (C) PRD-2: The map makes no mention of the popular vote at all, implying that it is completely unnecessary in determining who won the White House (American Government, 16th ed., pp. 335-336/Gateways to Democracy, 4th ed., pp. 336-343).

16. (C) LOR-1: Pluralists stress the importance of groups in American politics while elite theorists stress the importance of power and money. Remember, in comparison questions each wrong pair will still have at least one column correct (American Government, 16th ed., pp. 7-8/ Gateways to Democracy, 4th ed., pp. 12-13).

Page 42: Preparing for the AP U.S. Government and Politics Exam · 2020-03-20 · multiple-choice portion of the AP ... Congress could pass legislation providing stronger “whistleblower”

32 A DIAGNOSTIC TEST

© 2019 Cengage Learning, Inc. May not be scanned, copied or duplicated, or posted to a publicly accessible website, in whole or in part.

17. (A) LOR-2: Yoder held that requiring Amish students to attend school beyond the eighth grade inhibited their ability to meaningfully practice their religion. Engel found that a public school sponsoring prayer was equivalent to a government entity establishing a preference for one religion over another or for religion in general over non-religion. Remember, in comparison questions each wrong pair will still have at least one column correct (American Government, 16th ed., pp. 109-111/Gateways to Democracy, 4th ed., pp. 117-118).

18. (D) CON-3: Discharge petitions are unique to the House and the filibuster is unique to the Senate. Remember, in comparison questions each wrong pair will still have at least one column correct (American Government, 16th ed., p. 315/Gateways to Democracy, 4th ed., p. AP43).

19. (C) PMI-4: There are two clues here: one is that liberals tend to be more comfortable with government intervention than conservatives and the other is that liberals tend to place equality of opportunity higher on their list of priorities than conservatives. Remember, in comparison questions each wrong pair will still have at least one column correct (American Government, 16th ed., pp. 164-165/Gateways to Democracy, 4th ed., pp. 193-196).

20. (B) CON-6: The argument over affirmative action centers on whether or not it combats discrimination or promotes it. Those in favor say that it makes up for historic discrimination, while those opposed say that it just shifts the discrimination to white people. Remember, in comparison questions each wrong pair will still have at least one column correct (American Government, 16th ed., pp. 141-142/Gateways to Democracy, 4th ed., pp. 164-165).

21. (B) PRD-1: The passage discusses King’s reasoning for why protests were necessary. The oppression of black citizens had made their conventional political participation ineffective. This passage, as well as most of the Birmingham letter, were an explanation of the need for civil disobedience (American Government, 16th ed., pp. 257-258/Gateways to Democracy, 4th ed., pp. AP19-AP20).

22. (D) PMI-3: Literacy tests were outlawed by the VRA of 1965. Poll taxes are a tempting answer, but those were actually banned by constitutional amendment later on (American Government, 16th ed., pp. 135-136/Gateways to Democracy, 4th ed., pp. 155-156).

23. (B) MPA-3: The Fifteenth Amendment explicitly prohibits denying the right to vote on the basis of race (American Government, 16th ed., p. A13/Gateways to Democracy, 4th ed., p. 142).

24. (A) CON-3: The Shaw decision struck down racially-motivated congressional districts, even if they were theoretically for the benefit of African Americans (American Government, 16th ed., p. 301/Gateways to Democracy, 4th ed., p. AP44).

25. (D) LOR-2: Kennedy is using historical examples to illustrate that civil liberties have been trampled before, but that those instances were regretted after the fact (American Government, 16th ed., pp. 97-98 and 119/Gateways to Democracy, 4th ed., pp. 98-102).

26. (A) LOR-2: The safety of other citizens is a primary reason for the courts to allow the limiting of civil liberties. In the case of Kennedy’s speech, the threat of communism during the Cold War was frequently used to justify limiting those liberties (American Government, 15th ed., pp. 102 and 116/Gateways to Democracy, 4th ed., pp. 98-102).

27. (B) LOR-3: Since privacy is never mentioned by name, its inclusion among rights and liberties is controversial. Liberals and civil rights advocates, of which Kennedy was both, typically are in favour of recognizing the right to privacy (American Government, 16th ed., pp. 139-141/Gateways to Democracy, 4th ed., pp. 122-123).

Page 43: Preparing for the AP U.S. Government and Politics Exam · 2020-03-20 · multiple-choice portion of the AP ... Congress could pass legislation providing stronger “whistleblower”

A DIAGNOSTIC TEST 33

© 2019 Cengage Learning, Inc. May not be scanned, copied or duplicated, or posted to a publicly accessible website, in whole or in part.

28. (A) PMI-2: Power of the purse refers to Congress’ ability to use its control of the budget to reward or punish bureaucratic agencies. It is one of the few controls they have over those agencies (American Government, 16th ed., pp. 382-383/Gateways to Democracy, 4th ed., p. 405).

29. (D) CON-1: Amending the Constitution requires approval and both a state and national level. The process was deliberately designed to be federalist and difficult to do (American Government, 16th ed., pages 35 and 504).

30. (B) LOR-3: The McDonald decision held, for the first time, that the Second Amendment specifically referred to keeping firearms in private homes and that states were obligated to respect that right (American Government, 16th ed., pp. 100-102/Gateways to Democracy, 4th ed., pp. AP15-AP16).

31. (A) PRD-1: When the government is protecting citizens from discrimination, both from the government and from others, it falls under the umbrella of civil rights (American Government, 16th ed., p. 123/Gateways to Democracy, 4th ed., p. 136).

32. (D) CON-6: The Fourteenth Amendment consists of a due process clause and an equal protection clause. The equal protection clause is the one that has been consistently used to decide civil rights issues (American Government, 16th ed., pp. 99 and 503/Gateways to Democracy, 4th ed., pp. 136-137).

33. (B) CON-2: Block grants are grants handed out by the federal government with very few restrictions. States love them because they can spend the money any way they see fit as long as it fits under a broad category like education (American Government, 16th ed., p. 65/Gateways to Democracy, 4th ed., pp. 80 and AP11).

34. (D) MPA-2: Exit polls typically poll every tenth person leaving an election place. They do little to try to balance demographic factors like age, race, and party membership (American Government, 16th ed., p. 155/Gateways to Democracy, 4th ed., pp. 184-186).

35. (A) CON-5: Stare decisis roughly translates to “let it stand,” a direct reference to upholding precedent and using it to determine the ruling in the case at hand (American Government, 16th ed., p. 411/Gateways to Democracy, 4th ed., p. AP54).

36. (A) LOR-1: It is important to remember that the Declaration was mostly a list of complaints against the British king. It established some fundamental rights and concepts, such as popular sovereignty, but did little to establish a working form of government (American Government, 16th ed., pp. 22-24 and A1-A2/Gateways to Democracy, 4th ed., pp. 598-605).

37. (B) CON-1: One of the key weaknesses of the Articles was a lack of taxation power. This was done to respect individual liberty, but it created massive financial headaches for the country (American Government, 16th ed., pp. 25-26/Gateways to Democracy, 4th ed., pp. 32-33).

38. (A) CON-5: Article III of the Constitution outlines the judicial branch but it does so in very vague terms. It acknowledges the court system and the general outline thereof, but does little to grant actual powers. The Court essentially had to do that for itself (American Government, 16th ed., pp. 393, 400, and A8/Gateways to Democracy, 4th ed., pp. 517-518).

39. (D) PMI-2: An iron triangle consists of a bureaucratic agency, an interest group, and a congressional committee that are all connected to the same policy area. The three groups work to advance policies mutually beneficial to all of them. Members working for one of these groups often have past ties to another, creating fears of corruption and shady deals (American Government, 16th ed., pp. 381-382/Gateways to Democracy, 4th ed., pp. 503 and AP51).

40. (B) PMI-3: Title IX required school to fund extra-curricular activities for boys and girls equally. This had a massively improving effect on women’s athletics in particular (American Government, 16th ed., pp. 136-139/Gateways to Democracy, 4th ed., p. AP19).

Page 44: Preparing for the AP U.S. Government and Politics Exam · 2020-03-20 · multiple-choice portion of the AP ... Congress could pass legislation providing stronger “whistleblower”

34 A DIAGNOSTIC TEST

© 2019 Cengage Learning, Inc. May not be scanned, copied or duplicated, or posted to a publicly accessible website, in whole or in part.

41. (B) PMI-1: This option explains the process correctly. Make sure that you get the order (House, then Senate) correct as well as the fractional voting requirements (American Government, 15th ed., pp. 358-359/Gateways to Democracy, 4th ed., pp. 43 and 45).

42. (C) MPA-1: Political science studies have shown a general trend that all members of a particular generation become slightly more conservative with age (American Government, 16th ed., pp. 158-163/Gateways to Democracy, 4th ed., pp. 189-190).

43. (D) PMI-2: The merit system is intended to make it so that bureaucratic positions are awarded purely on talent and qualifications rather than political or social connections (American Government, 16th ed., p. 374/Gateways to Democracy, 4th ed., pp. 496-497).

44. (A) PMI-5: Recruiting candidates is one of the primary functions of a political party. Parties would prefer that no race, no matter how small, goes uncontested. Parties will also help with fundraising and strategy to entice more people to run (American Government, 16th ed., p. 197/Gateways to Democracy, 4th ed., pp. 287-289).

45. (B) PMI-2: Congressional oversight joins power of the purse as one of the ways in which Congress can regulate the bureaucracy. With it, Congress ensures that agencies are acting within the law and doing what they are supposed to be doing. (American Government, 15th ed., p. 382/Gateways to Democracy, 4th ed., pp. 409-410).

46. (D) CON-5: In Marbury v. Madison, the Supreme Court asserted its authority to invalidate state and federal laws that violate the Constitution (American Government, 16th ed., pp. 393-394/Gateways to Democracy, 4th ed., p. 518).

47. (B) PMI-5: Economic resources are critical to the success of an interest group, perhaps more than anything else. This money allows them to donate to campaigns, to research their specialty areas, and hire consultants and experts (American Government, 16th ed., pp. 408–411/Gateways to Democracy, 4th ed., p. 279).

48. (C) CON-3: Divided government is when the president and at least one house of Congress are of opposite political parties. In this event the president’s legislative agenda and nominees are subjected to greater scrutiny, which results in delays (American Government, 16th ed., p. 317/Gateways to Democracy, 4th ed., p. 451).

49. (A) PRD-2: PACs are groups that are formed with the explicit purpose of collecting money and using it for political activity. Most of that money gets donated to campaigns or spent on “issue ads,” which advocate for a policy position but do not coordinate with any official campaigns (American Government, 16th ed., p. 229/Gateways to Democracy, 4th ed., pp. 336-338).

50. (C) CON-2: McCulloch is most notable for its firm affirmation of national supremacy when it comes to legal matters (American Government, 16th ed., p. 55/ Gateways to Democracy, 4th ed., pp. 75 and AP11).

51. (D) CON-2: The ability for local law enforcement to refuse enforcement of federal law is indicative of federalism. There are multiple layers of government, with multiple layers of laws, and each layer must enforce its own laws (American Government, 16th ed., pp. 57-59/Gateways to Democracy, 4th ed., pp. 78-80).

52. (B) PMI-4: Supply-side economists favor less government planning, more deregulation, and tax cuts to stimulate economic growth (American Government, 16th ed., p. 459/Gateways to Democracy, 4th ed., p. 199).

53. (D) PMI-4: Monetary policy is concerned with the amount of money in circulation. This amount is manipulated to control its value. The Board of Governors of the Federal Reserve is charged with setting monetary policy (American Government, 16th ed., pp. 461-462/Gateways to Democracy, 4th ed., pp. 577-578).

Page 45: Preparing for the AP U.S. Government and Politics Exam · 2020-03-20 · multiple-choice portion of the AP ... Congress could pass legislation providing stronger “whistleblower”

A DIAGNOSTIC TEST 35

© 2019 Cengage Learning, Inc. May not be scanned, copied or duplicated, or posted to a publicly accessible website, in whole or in part.

54. (A) MPA-2: Polls tend to command media attention and, while scientific polling can be reasonably accurate, they do little to actually examine the true substance of the issues. Voters then just care about who is winning rather than how or why they are winning (American Government, 16th ed., pp. 154 and 277/Gateways to Democracy, 4th ed., pp. AP25-AP26).

55. (B) CON-4: The Twenty-second Amendment was passed after Roosevelt’s death to ensure no other President would have more than two terms. Prior to Roosevelt, Presidents stepped aside after two terms simply out of tradition (American Government, 16th ed., pp. 332-333 and A15-A16/Gateways to Democracy, 4th ed., p. 441).

ANSWERS TO THE FREE-RESPONSE QUESTIONS:

QUESTION 1 For free response questions you will receive points for doing what the question asks or commands. The point is all-or-nothing, meaning that you either get credit for it or you do not. Part (a): One point is awarded for correctly describing one thing Congress could do in response to the movement. To receive credit, the answer must address the concerns of the movement AND be something that Congress has the constitutional authority to do. Possible answers would include strengthening labor laws to protect employees who report their bosses and increasing the scope of the bureaucracy to better regulate sexual misconduct in the workplace. Be careful that you do not focus on laws that are state matters rather than federal. (American Government, 16th ed., p. 292/Gateways to Democracy, 4th ed., pp. 405-410).

Part (b): One point is awarded for connecting the action above to Congress’ relationship with the President. In this description you can focus on the President helping OR hindering the action; you do not have to do both. The best answer is that the President will quickly sign any bill Congress passes or use his media bully pulpit to speak out on the issue, if he is friendly with Congress. If the President is adversarial with Congress, he may veto their legislation (American Government, 16th ed., pp. 329-330/Gateways to Democracy, 4th ed., pp. 428 and 451).

Part (c): One point is awarded for connecting the movement to a potential campaign strategy. This must both relate to the movement and actually have a viable reason for impacting the outcome of the election. For example, a major party may cater more towards women’s issues since the movement has likely raised awareness about sex discrimination (American Government, 16th ed., pp. 215–219/Gateways to Democracy, 4th ed., pp. 340-341).

QUESTION 2 Part (a): The AP® exam will usually begin quantitative stimulus questions by asking you a generic question about the chart as a whole, to make sure you understand it. For this part, one point is awarded for describing what the chart seems to be telling you. Based on how the chart has been laid out, it is clearly emphasizing that most Americans favor some legal path for immigrants to remain in the country. This is evident both in how the chart was designed and in the overwhelming majority of responses (American Government, 16th ed., pp. 162 and 168/Gateways to Democracy, 4th ed., pp. 566-570).

Part (b): One point is awarded for connecting the chart to the political ideologies of the two major parties. To receive credit, your answer must acknowledge the pre-existing dispositions of the two major parties. Democrats are typically more pro-immigrant and, since the public apparently supports that, they should move that issue to the top of their platform. Republicans are typically more suspicious of immigrants,

Page 46: Preparing for the AP U.S. Government and Politics Exam · 2020-03-20 · multiple-choice portion of the AP ... Congress could pass legislation providing stronger “whistleblower”

36 A DIAGNOSTIC TEST

© 2019 Cengage Learning, Inc. May not be scanned, copied or duplicated, or posted to a publicly accessible website, in whole or in part.

particularly those here illegally. Since the public seems sympathetic to immigrants, Republicans may want to re-think their position or at least downplay it (American Government, 16th ed., p. 195/Gateways to Democracy, 4th ed., pp. 314-316).

Part (c): One point is awarded for EACH of the two influences that you explain. To receive points, you must specifically define the term and connect it to the chart at hand. Sample sizes must be large enough to be representative of the population as a whole. If the sample size is too small, the poll is not an accurate reflection of popular opinion. A large margin of error can make close polling numbers invalid. Margins of error need to be around three or four percent to be considered a quality survey. If questions are phrased in a certain way they may lead the respondent. In this case, questions that make immigration sound positive and immigrants as nothing but nice, hard-working people, could bias respondents into supporting immigration (American Government, 16th ed., pp. 154-156, 162 and 168/Gateways to Democracy, 4th ed., pp. 183-188).

QUESTION 3 Part (a): One point is awarded for correctly identifying that both cases should do with the free exercise and establishment clauses of the First Amendment. Freedom of religion would also be an acceptable answer (American Government, 16th ed., pp. 108-111/Gateways to Democracy, 4th ed., pp. 117-120).

Part (b): One point is awarded for correctly identifying that Smith held that the prohibition of peyote did NOT fundamentally inhibit the exercise of religion, while requiring Amish children to attend school did. The Court held that both laws were not specifically targeting either religion, but felt that the Native American religious practices could still take place without peyote. Attending school beyond the eighth grade, meanwhile, is considered prideful among the Amish and is therefore a sin. The law would be requiring Amish children to do something their religion considers inappropriate. (American Government, 16th ed., pp. 108–111/Gateways to Democracy, 4th ed., pp. 117-120).

Part (c): One point is awarded for describing a viable option for Native American interest groups. To receive credit, you answer should reflect an awareness of interest group politics and how they function in the United States. In other words, mention a specific action that demonstrates you have an awareness of how interest groups achieve their goals. Good answers would include pressuring voters to support the Native American cause, gaining access to state legislators to express their opinion, and drafting potential legislation that would legalize peyote for religious use (American Government, 16th ed., pp. 248-258/Gateways to Democracy, 4th ed., pp. 259-269).

QUESTION 4 One point is awarded for making a defensible claim or thesis that addresses the prompt. To earn a point, you must take a stance on the issue and do more than just restate the question at hand. For example, if you were arguing that judicial activism does overstep the intended power of the judicial branch, you would not receive credit for merely writing, “Judicial activism is overstepping the intended power of the courts.” Instead, you should write something more along the lines of, “Judicial activism is overstepping the intended power of the courts because there is no mention of such powers in the Constitution and it thwarts the idea of popular rule” (American Government, 16th ed., p. 394/Gateways to Democracy, 4th ed., pp. 530-531)

One point is awarded for EACH piece of evidence used in your argumentation. Evidence usage does not require a direct quote, but you should show enough awareness of the document in question to illustrate that you have read it. For example, citing Article III as the basis for the judicial branch in the U.S. Constitution

Page 47: Preparing for the AP U.S. Government and Politics Exam · 2020-03-20 · multiple-choice portion of the AP ... Congress could pass legislation providing stronger “whistleblower”

A DIAGNOSTIC TEST 37

© 2019 Cengage Learning, Inc. May not be scanned, copied or duplicated, or posted to a publicly accessible website, in whole or in part.

is a good idea. You can choose any document for your second piece of evidence, but sticking to the required foundational documents is a great way to ensure that your AP® grader will be familiar with the evidence in question (American Government, 16th ed., pp. A3-A17/Gateways to Democracy, 4th ed., pp. 606-622).

One point is awarded for connecting your evidence to your argument. You must make sure that the evidence you choose falls in line with your thesis. If you look at the sample thesis above, you will notice that it already mentions the Constitution, setting you up for an easy connection later. The AP® exam sets aside forty minutes for this essay, in part so you can plan and outline what you want to say. Take advantage of that time! (American Government, 16th ed., pp. 394 and A3-A17/Gateways to Democracy, 4th ed., pp. 606-622).

One point is awarded for acknowledging an opposing viewpoint and addressing it. This is an all-or-nothing point, so you should both come up with a credible viewpoint AND deal with it. If you fail to do one or the other, you will not earn any points. Any opposing viewpoint will do, so long as it is plausible. Once you have stated that viewpoint, you can either refute it (prove it wrong), concede to it (accept it as valid), or offer a rebuttal (accept the argument but counter with one of your own). If you refute it, it is a good idea to use evidence or something concrete to back up your assertion. If you concede it, make sure to explain why it doesn’t undermine your argument. For example, if you were in favor of judicial activism, you might say something like, “Critics of this view would maintain that judicial activism is undemocratic. This is indeed a concern, as the Supreme Court is not elected and justices serve for life, and the Court should be aware of the potential consequences of its power.” In that example the argument was conceded but the author did not let it undermine her/his original assertion. Finally, if you rebut the argument, be sure to throw in evidence of your own. Rebuttal is different from refutation because you are admitting the counterargument is valid, but then proving why it is not a concern. For example: “Judicial activism does indeed come across as undemocratic. However, decisions from an institution whose members are not elected is necessary to protect the rights of minorities, both social and numerical” (American Government, 16th ed., pp. 394 and A3-A17/Gateways to Democracy, 4th ed., pp. 606-622).

Full Sample Answers for Free Response Questions 1. This is a concept application style question, as described in the Introduction

section of this book. (a) Congress could pass legislation providing stronger “whistleblower”

legislation, aimed at protecting women who come forward with allegations against their superiors in the workplace.

(b) If the President and both houses of Congress are controlled by the same party, then it will be much easier to pass whistleblower legislation. It is much less likely that a President would veto legislation passed by his fellow party members.

(c) The #MeToo movement centers on women’s rights and the equal treatment of women, both socially and professionally. The popularity of the movement means that both political parties will have to show they embrace these concepts. Women’s issues and women’s rights will be featured more prominently in major party platforms for the next election.

2. This is a quantitative analysis style question, as described in the Introduction section of this book. (a) The overall message of the pie chart is that most Americans favor some

path to legal residency for immigrants. The chart points out that citizenship eligibility was the most popular choice and 71% of respondents favor some kind of option that allows the immigrant to remain in the country legally.

Page 48: Preparing for the AP U.S. Government and Politics Exam · 2020-03-20 · multiple-choice portion of the AP ... Congress could pass legislation providing stronger “whistleblower”

38 A DIAGNOSTIC TEST

© 2019 Cengage Learning, Inc. May not be scanned, copied or duplicated, or posted to a publicly accessible website, in whole or in part.

(b) NOTE: All three choices have been explained below for the purposes of this sample answer. You will only need to answer two of the three options to receive full credit

i. The sample size for this survey would have to be large enough to accurately reflect the nation as a whole, which means approximately 1500 responses are needed. If the sample size is too small the pollster cannot claim that it accurately reflects public opinion.

ii. Even the most accurate polls are going to have a margin of error. If this margin of error is +/- 5%, it means that all numbers presented in the chart could sway either way by that many points. While the chart says that 43% favor citizenship eligibility, that number could actually be as high as 48% or as low as 38%.

iii. The questions used in the survey are not divulged along with the chart. If the questions were written in a way that could potentially guide the respondent, it throws off all the results. For example, asking “Do hard-working immigrants seeking a better life in America deserve a path to citizenship?” would drastically increase the number of people who would be in support of such a measure

(c) NOTE: Sample answers for either party are included here. You only needed to write about one or the other. Democrats are typically viewed as being more favorable toward immigrants than Republicans. Since most of the nation favors some sort of legal residency for immigrants, the Democrats should tout their support of immigrants and make it a leading part of their platform. This would include a simpler path to citizenship and expedited green card processing. These would both provide legal paths to citizenship while ensuring that the existing process is still respected.

Republicans are typically viewed as being more suspicious of immigrants. However, they could use this information to stress that they are comfortable with legal migration and that their concerns focus more on illegal immigration. Republicans could build a platform around immigration reform that would provide a path to citizenship for immigrants currently in the country while also making it harder to enter the country illegally. This move would satisfy traditional Republicans while also acknowledging the results of the poll.

3. This is a Supreme Court comparison style question, as described in the Introduction section of this book. (a) The cases of Employment Division v. Smith (1990) and Wisconsin v. Yoder

(1972) both center on the free exercise clause of the First Amendment. (b) In Smith, the Court found that the law was not aimed specifically at Native

Americans and denying them unemployment benefits did not prevent them from practicing their religion. In Yoder, on the other hand, requiring Amish children to attend school beyond the eighth grade did explicitly violate the values of their religious community. The law made it impossible for Amish children to both be true to their faith and abide by the law.

(c) Native American groups could lobby Oregon state legislators to legalize peyote for religious purposes while simultaneously raising awareness of the issues with voters. Legislators could make a variety of religious exemptions for peyote use, including unemployment benefits. Likewise, presenting this as religious discrimination towards an historically oppressed minority would likely build significant support among voters and increase pressure on state legislators.

4. This is an argumentative essay style question, as described in the Introduction section of this book. (a) Judicial activism oversteps the intended power of the judicial branch

because it effectively turns the courts into a legislative body and it is insufficiently checked by the other powers.

Page 49: Preparing for the AP U.S. Government and Politics Exam · 2020-03-20 · multiple-choice portion of the AP ... Congress could pass legislation providing stronger “whistleblower”

A DIAGNOSTIC TEST 39

© 2019 Cengage Learning, Inc. May not be scanned, copied or duplicated, or posted to a publicly accessible website, in whole or in part.

(b) Article III of the U.S. Constitution is rather vague on the role of the judicial branch, focusing more on the terms of judges and justices and the jurisdiction of the courts rather than the specifics of its power. This lack of specificity should not be taken as an open invitation to expand the courts’ role in government, however. The concept of limited government implies that government branches should be restrained to the powers expressly delegated to them. Article I, meanwhile, clearly grants Congress the power to make laws necessary and proper for governing the land. Judicial activism partially strips Congress of this power and effectively allows judges to make policy from the judicial bench.

(c) Alexander Hamilton furthered this line of reasoning in Federalist Paper No. 78. Hamilton acknowledged that the Supreme Court had the power to review the constitutionality of laws, but he also acknowledged that the separation of powers had left the courts weak enough that their powers of judgment could not infringe upon the express powers of the other branches. Judicial activism flies in the face of this notion by over-extending judicial power and influence. By allowing the courts to dictate policy, it makes them stronger than the Founders intended and leads to policy being generated by the least democratic of the three branches.

NOTE: Below you will find samples for refutation, concession, and rebuttal. They are presented here as a demonstration for what each would look like. You would only need to do ONE in your essay.

Refutation: Supporters of judicial activism would claim that it is merely an extension of the powers found in judicial review. This line of reasoning maintains that the Court is merely clarifying its position and how that position needs to be implemented. While it is tempting to believe that this is the case, the need to clarify a ruling should not come at the expense of the separation of powers. The courts may very well identify when laws are not constitutional and they should indeed monitor the behavior of the other two branches, but there is a difference between identifying unconstitutional behavior and enacting policy. The Court can fulfill the responsibility of judicial review without making its own policy decisions.

Concession: Supporters of judicial activism would contend that such behavior is necessary to protect minority rights. The argument goes that minorities are unable to sufficiently impact the democratically-elected branches and are thus reliant upon the courts to protect their rights. Denying the courts the option of judicial activism leaves them without a way to ensure the interests of these people. This is a valid concern, as the courts have historically been the best safeguard of minority rights and reducing judicial activism would certainly limit them in this capacity. Restraining or eliminating the practice of judicial activism could potentially lead to negative consequences for minorities in the United States.

Rebuttal: Proponents of judicial activism assert that the practice is a necessary block against a tyranny of the majority. As the only branch not beholden to re-election, judges and justices can decide cases based on merits alone and can thus concern themselves with what is just rather than what is popular. This was indeed the intent of the Framers, to have a branch almost wholly insulated from popular opinion, but that insulation cannot come at the cost of crucial constitutional principles such as separation of powers. Judicial activism extends beyond protection against tyranny, as simple judicial review can accomplish that. Instead, activism directly treads on the expressed power of another branch. Upholding one of the Founders’ principles (preventing tyranny) cannot come at the cost of another (separation of powers).

Page 50: Preparing for the AP U.S. Government and Politics Exam · 2020-03-20 · multiple-choice portion of the AP ... Congress could pass legislation providing stronger “whistleblower”

40 A DIAGNOSTIC TEST

© 2019 Cengage Learning, Inc. May not be scanned, copied or duplicated, or posted to a publicly accessible website, in whole or in part.

CALCULATING YOUR SCORE ON THE AP® EXAM:

SECTION 1: MULTIPLE-CHOICE QUESTIONS

Total number answered correctly _________

SECTION II: FREE-RESPONSE QUESTIONS

Remember that each question is weighted equally. Some quick arithmetic is necessary to adjust for the fact that different questions have different point values. Question 1 _________ × 20 equals _________ Out of 3 do not round Question 2 _________ × 15 equals _________ Out of 4 do not round Question 3 _________ × 20 equals _________ Out of 3 do not round Question 4 _________ × 12 equals _________ Out of 5 do not round Total For Section II _________

COMPOSITE SCORE: _________ + _________ = _________ Section I Section II Composite Score

Student scores are weighted differently each year to determine the final AP® score. The AP® exam is curved based on national average performance. The conversion chart below is relatively accurate to the average weighting range on the exam.

COMPOSITE SCORE RANGE AP® GRADE 155–200 5 137–154 4 110–136 3 80–135 2 0–79 1